Anda di halaman 1dari 16

GUIJARNO VS.

CIR, 52 S 307

happen though that whatever group may be in control of the


organization may simply ignore his most-cherished desires and treat
him as if he counts for naught. The antagonism between him and the
group becomes marked. Dissatisfaction if given expression may be
labeled disloyalty. In the labor field, the union under such
circumstances may no longer be a haven of refuge, but indeed as
much of a potential foe as management itself.

Facts: Nineteen employees were dismissed by Central Santos Lopez


Co., Inc upon their expulsion from United Sugar Workers Union-ILO.
The sugar company assumed that it had to dismiss the workers by
virtue of the closed-shop provision in the then existing collective
bargaining agreement which states that laborers who are no longer
members of good standing in the union may be dismissed by the
respondent company if their dismissal is sought by the union. It
argued that it has never committed any acts of unfair labor practice
since the union sought the dismissal of the said workers; that it has a
solemn obligation to comply with the terms and conditions of the
contract; and that a closed-shop agreement is sanctioned under this
jurisdiction for such kind of agreement is expressly allowed under the
provisions of Republic Act 875 known as the Industrial Peace Act and
the dismissal of complainants is merely an exercise of a right allowed
by said law. It was established that the terminated workers were
employed long before the collective bargaining contract has been
entered into. The lower courts held that the dismissal was justifiable
under the closed-shop provision of the collective bargaining
agreement. Hence, this petition for review.

ELECTROMAT MANUFACTURING & RECORDING CORPORATION


VS. HON. LAGUNZAD, G.R. NO. 172699, JULY 27, 2011
FACTS: The private respondent Nagkakaisang Samahan ng
Manggagawa ng ElectromatWasto (union), a charter affiliate of the
Workers Advocates for Struggle, Transformation and Organization
(WASTO), applied for registration with the Bureau of Labor Relations
(BLR). The BLR thereafter issued the union a Certification of Creation
of Local Chapter (equivalent to the certificate of registration of an
independent union), pursuant to Department Order No. (D.O.) 4003.
On October 1, 2003, the petitioner Electromat Manufacturing and
Recording Corporation (company) filed a petition for cancellation of the
union's registration certificate, for the union's failure to comply with
Article 234 of the Labor Code. It argued that D.O. 4003 is an
unconstitutional diminution of the Labor Code's union registration
requirements under Article 234. Specifically, it assails as
unconstitutional Section 2(E), Rule III of D.O. 4003 which provides:
The report of creation of a chartered local shall be accompanied by a
charter certificate issued by the federation or national union indicating
the creation or establishment of the chartered local.

Issue: Whether or nor a closed-shop provision in a collective


bargaining contract is retroactive
Held: NO. In Confederated Sons of Labor v. Anakan Lumber Co., it
was said that "In order that an employer may be deemed bound, under
a collective bargaining agreement, to dismiss employees for non-union
membership, the stipulation to this effect must be so clear and
unequivocal as to leave no room for doubt thereon. An undertaking of
this nature is so harsh that it must be strictly construed, and doubts
must be resolved against the existence of "closed shop"." Furthermore,
it was stated in Freeman Shirt Manufacturing v. Court of Industrial
Relations that "The closed-shop agreement authorized under sec. 4,
subsec. a(4) of the Industrial Peace Act applies to persons to be hired
or to employees who are not yet members of any labor organization. It
is inapplicable to those already in the service who are members of
another union. To hold otherwise, i. e., that the employees in a
company who are members of a minority union may be compelled to
disaffiliate from their union and join the majority or contracting union,
would render nugatory the right of all employees to self-organization
and to form, join or assist labor organizations of their own choosing, a
right guaranteed by the Industrial Peace Act as well as by the
Constitution."

DECISION OF LOWER COURTS: * DOLE: dismissed the petition. *


BLR Director: affirmed the decision of the DOLE. * CA: affirmed BLR
Director.
ISSUE: WON the Department order is constitutional or whether D.O.
4003 is a valid exercise of the rulemaking power of the DOLE.
APPLICABLE LAWS: Article 234 states:
ART. 234. Requirements of Registration.[13] Any applicant labor
organization, association or group of unions or workers shall acquire
legal personality and shall be entitled to the rights and privileges
granted by law to legitimate labor organizations upon issuance of the
certificate of registration based on the following requirements:
(a) Fifty pesos (P50.00) registration fee;
(b) The names of its officers, their addresses, the principal address of
the labor organization, the minutes of the organizational meetings and
the list of the workers who participated in such meetings;
(c) The names of all its members comprising at least twenty percent
(20%) of all the employees in the bargaining unit where it seeks to
operate;
(d) If the applicant union has been in existence for one or more years,
copies of its annual financial reports; and
(e)
Four (4) copies of the constitution and bylaws of the applicant
union, minutes of its adoption or ratification, and the list of the
members who participated in it.

According to the Court, the creation of labor unions is a means of


assuring that fundamental constitutional objectives such as the
protection of labor (full employment and equality in employment,
ensure equal work opportunities regardless of sex, race, or creed, and
regulate the relations between workers and employers) and the
promotion of social justice (dignity, welfare, and security of all the
people) would be achieved. It is the instrumentality through which an
individual laborer who is helpless as against a powerful employer may,
through concerted effort and activity, achieve the goal of economic
well-being. That is the philosophy underlying the Industrial Peace Act.
For, rightly has it been said that workers unorganized are weak;
workers organized are strong. Necessarily then, they join labor unions.
To further increase the effectiveness of such organizations, a closedshop has been allowed. It could happen, though, that such a stipulation
which assures further weight to a labor union at the bargaining table
could be utilized against minority groups or individual members
thereof. There are indications that such a deplorable situation did so
manifest itself here. The lower courts were not sufficiently alert to such
a danger. What is worse, it paid no heed to the controlling doctrine
which is merely a recognition of a basic fact in life, namely, that power
in a collectivity could be the means of crushing opposition and stifling
the voices of those who are in dissent. The right to join others of like
persuasion is indeed valuable. An individual by himself may feel
inadequate to meet the exigencies of life or even to express his
personality without the right to association being vitalized. It could

RULING: YES, constitutional.


D.O. 4003 represents an expression of the government's
implementing policy on trade unionism. It builds upon the old rules by
further simplifying the requirements for the establishment of locals or
chapters. As in D.O. 9, we see nothing contrary to the law or the
Constitution in the adoption by the Secretary of Labor and Employment
of D.O. 4003 as this department order is consistent with the intent of
the government to encourage the affiliation of a local union with a
federation or national union to enhance the local's bargaining power. If
changes were made at all, these were those made to recognize the
distinctions made in the law itself between federations and their local
chapters, and independent unions; local chapters seemingly have
lesser requirements because they and their members are deemed to
be direct members of the federation to which they are affiliated, which
federations are the ones subject to the strict registration requirements
of the law.

15

MARIWASA SIAM CERAMICS, INC VS. SECRETARY OF LABOR,


GR. NO. 183327, DEC 21, 2009

locals, and after having submitted the requirements to BLR, granted


the separate certification elections.
CA affirmed Secretary.

FACTS: On May 2005, private respondent Samahan Ng Mga


Manggagawa Sa Mariwasa Siam Ceramics, Inc. (SMMSCIndependent) was issued a Certificate of Registration as a legitimate
labor organization by the Department of Labor and Employment
(DOLE), Region IV-A.
On June 2005, petitioner Mariwasa Siam Ceramics, Inc. filed a Petition
for Cancellation of Union Registration against private respondent,
claiming that the latter violated Article 234 of the Labor Code for not
complying with the 20% requirement and that it committed massive
fraud and misrepresentation in violation of Article 239 of the same
code.
The Regional Director of DOLE IV-A issued an Order granting the
petition, revoking the registration of respondent, and delisting it from
the roster of active labor unions.
SMMSC-Independent appealed to the Bureau of Labor Relations. BLR
ruled in favor of the respondent, thus, they remain in the roster of
legitimate labor organizations.
The petitioner appealed and insisted that private respondent failed to
comply with the 20% union membership requirement for its registration
as a legitimate labor organization because of the disaffiliation from the
total number of union members of 102 employees who executed
affidavits recanting their union membership.
Hence, this petition for review on certiorari under Rule 45 of the Rules
of Court.

Issues: (1) Whether or not BLR is the only entity that may issue
registrations. NO
(2) Whether or not supervisory and RnF may file separate certification
elections. YES
(3) Whether or not respondents engaged in commingling. NO.
Held: Petitioner alleges only BLR may issue registration. However, Art.
235 states the BLR and/or the Regional Offices may do such. Even
after the amendment the Regional Offices were not divested of their
power, but only that the BLR will act on such registration made to the
Regional Offices.
Section 5. Effect of registration The labor organization or workers
association shall be deemed registered and vested with legal
personality on the date of issuance of its certificate of registration.
Such legal personality cannot thereafter be subject to collateral attack,
but maybe questioned only in an independent petition for cancellation
in accordance with these Rules.
APSOTEU is a legitimate organization and may validly issue a charter
to its affiliates, and this stays in effect until cancelled or revoked, and
until then, possess a separate legal personality from each other., even
with the commonalities.
A local union does not owe its existence to the federation with which it
is affiliated. It is a separate and distinct voluntary association owing its
creation to the will of its members. Mere affiliation does not divest the
local union of its own personality, neither does it give the mother
federation the license to act independently of the local union. Hence,
local unions are considered principals while the federation is deemed
to be merely their agent. As such principals, the unions are entitled to
exercise the rights and privileges of a legitimate labor organization.

ISSUES: 1) Whether or not there was failure to comply with the 20%
union membership requirement
2) Whether or not the withdrawal of 31 union members affected the
petition for certification election insofar as the 30% requirement is
concerned
RULING: The Supreme Court DENIED the petition.
On the first issue, while it is true that the withdrawal of support may be
considered as a resignation from the union, the fact remains that at the
time of the unions application for registration, the affiants were
members of respondent and they comprised more than the required
20% membership for purposes of registration as a labor union. Article
234 of the Labor Code merely requires a 20% minimum membership
during the application for union registration. It does not mandate that a
union must maintain the 20% minimum membership requirement all
throughout its existence.
On the second issue, it appears undisputedly that the 31 union
members had withdrawn their support to the petition before the filing of
said petition. The distinction must be that withdrawals made before the
filing of the petition are presumed voluntary unless there is convincing
proof to the contrary, whereas withdrawals made after the filing of the
petition are deemed involuntary. Therefore, following jurisprudence, the
employees were not totally free from the employers pressure and so
the voluntariness of the employees execution of the affidavits
becomes suspect.
The cancellation of a unions registration doubtless has an impairing
dimension on the right of labor to self-organization. For fraud and
misrepresentation to be grounds for cancellation of union registration
under the Labor Code, the nature of the fraud and misrepresentation
must be grave and compelling enough to vitiate the consent of a
majority of union members.

PROGRESSIVE DEVELOPMENT CORPORATION PIZZA HUT VS.


LAGUESMA, 271 SCRA 593
FACTS: On July 9, 1993, Nagkakaisang Lakas ng Manggagawa
(NLM)-Katipunan (respondent Union) filed a petition for certification
election with the Department of Labor (National Capital Region) in
behalf of the rank and file employees of the Progressive Development
Corporation (Pizza Hut).
Petitioner filed on August 20, 1993, a verified Motion to Dismiss the
petition alleging fraud, falsification and misrepresentation in the
respondent. Unions registration making it void and invalid: a)
respondent Unions registration was tainted with false, forged, double
or multiple signatures of those who allegedly took part in the ratification
of the respondent Unions constitution and by-laws and in the election
of its officers that there were two sets of supposed attendees to the
alleged organizational meeting that was alleged to have taken place on
June 26, 1993; that the alleged chapter is claimed to have been
supported by 318 members when in fact the persons who actually
signed their names were much less; and b) while the application for
registration of the charter was supposed to have been approved in the
organizational meeting held onJune 27, 1993, the charter certification
issued by the federation KATIPUNAN was datedJune 26, 1993or one
(1) day prior to the formation of the chapter, thus, there were serious
falsities in the dates of the issuance of the charter certification and the
organization meeting of the alleged chapter.
On August 30, 1993, petitioner filed a Petition seeking the cancellation
of the Unions registration on the grounds of fraud and falsification,
docketed as BIR Case No. 8-21-83. Motion was likewise filed by
petitioner with the Med-Arbiter requesting suspension of proceedings
in the certification election case until after the prejudicial question of
the Unions legal personality is determined in the proceedings for
cancellation of registration.
However, in an Order dated September29, 1993,6Med-Arbiter Rasidali
C. Abdullah directed the holding of a certification election among
petitioners rank and file employees.
An appeal to the office of the Secretary of Labor, Labor Undersecretary
Bienvenido E. Laguesma in a Resolution dated December 29, 1993
denied the same.
Motion for reconsideration of the public respondents resolution was
denied.

COASTAL SUBIC BAY TERMINAL VS. DOLE, GR NO. 157117


Facts: Petitioners rank and file union and supervisors union both filed
for certification elections with the Med Arbiter.
RnF union insists it is registered by ALU and the Supervisor union
states it is registered with Associated Professional, Supervisory, Office
and Technical Union (APSOTEU).
Petitioner alleges both are not legitimate and proposed bargaining
units were not organized.
Med Arbiter dismissed both petitions and stated both organizations
belonged to the same federation.
Secretary reversed stating both have separate legal personalities and
separate certificates of registration from DOLE, and different sets of

15

On 27 July 1998, petitioner filed a motion to dismiss the petition for


certification election on the sole ground that herein respondent is not
listed or included in the roster of legitimate labor organizations based
on the certification issued by the Officer-In-Charge, Regional Director
of the DOLE Regional Office.
Undersecretary Baldoz concluded that respondent acquired legal
personality as early as 15 June 1998, the date it submitted the required
documents, citing Section 3, Rule VI of the New Rules Implementing
the Labor Code (Implementing Rules) which deems that a
local/chapter acquires legal personality from the date of filing of the
complete documentary requirements as mandated in the Implementing
Rules.
These two conclusions of the DOLE were affirmed in the assailed
Decision of the Court of Appeals.

ISSUE1: whether or not, after the necessary papers and documents


have been filed by a labor organization, recognition by the Bureau of
Labor Relations merely becomes a ministerial function.
RULING1: Art. 234. Requirements of registration. Any applicant
labor organization, association or group of unions or workers shall
acquire legal personality and shall be entitled to the rights and
privileges granted by law to legitimate labor organizations upon
issuance of the certificate of registration based on the following
requirements: (a) Fifty pesos (P50.00) registration fee; (b) The names
of its officers, their addresses, the principal address of the labor
organization, the minutes of the organizational meetings and the list of
the workers who participated in such meetings; (c) The names of all its
members comprising at least twenty percent (20%) of all the
employees in the bargaining unit where it seeks to operate; (d) If the
applicant union has been in existence for one or more years, copies of
its annual financial reports; and (e) Four (4) copies of the constitution
and by-laws of the applicant union, minutes of its adoption or
ratification, and the list of the members who participated in it.
A more than cursory reading of the aforecited provisions clearly
indicates that the requirements embodied therein are intended as
preventive measures against the commission of fraud. After a labor
organization has filed the necessary papers and documents for
registration, it becomes mandatory for the Bureau of Labor Relations to
check if the requirements under Article 234 have been sedulously
complied with. falsification and serious irregularities, especially those
appearing on the face of the application and the supporting
documents, a labor organization should be denied recognition as a
legitimate labor organization. And if a certificate of recognition has
been issued, the propriety of the labor organizations registration could
be assailed directly through cancellation of registration proceedings in
accordance with Articles 238 and 239 of the Labor Code, or indirectly,
by challenging its petition for the issuance of an order for certification
election.
Such requirements are a valid exercise of the police power, because
the activities in which labor organizations, associations and unions of
workers are engaged directly affect the public interest and should be
protected.
The employer needs the assurance that the union it is dealing with is a
bona fide organization, one which has not submitted false statements
or misrepresentations to the Bureau. Clearly, fraud, falsification and
misrepresentation in obtaining recognition as a legitimate labor
organization are contrary to the Med-Arbiters conclusion not merely
collateral issues. The invalidity of respondent Unions registration
would negate its legal personality to participate in certification election.
Once a labor organization attains the status of a legitimate labor
organization it begins to possess all of the rights and privileges granted
by law to such organizations.
Inasmuch as the legal personality of respondent Union had been
seriously challenged, it would have been more prudent for the MedArbiter and public respondent to have granted petitioners request for
the suspension of proceedings in the certification election case, until
the issue of the legality of the Unions registration shall have been
resolved. Failure of the Med-Arbiter and public respondent to heed the
request constituted a grave abuse of discretion.

Issue: Whether or not respondent has acquired legal personality.


Held: YES. It could be properly said that at the exact moment
respondent was filing the petition for certification, it did not yet possess
any legal personality, since the requisites for acquisition of legal
personality under Section 3, Rule VI of Department Order No. 9 had
not yet been complied with. It could also be discerned that the intention
of the Labor Code and its Implementing Rules that only those labor
organizations that have acquired legal personality are capacitated to
file petitions for certification elections. Such is the general rule. Yet
there are peculiar circumstances in this case that allow the Court to
rule that respondent acquired the requisite legal personality at the
same time it filed the petition for certification election. In doing so, the
Court acknowledges that the strict letter of the procedural rule was not
complied with. However, labor laws are generally construed liberally in
favor of labor, especially if doing so affirms the constitutionally
guaranteed right to self-organization.
Under Section 3, Rule VI of Department Order No. 9, it is the
submission of these same documents to the Regional Office or Bureau
that operates to vest legal personality on the local/chapter. There is no
doubt that on 15 June 1998, or the date respondent filed its petition for
certification election, attached thereto were respondents constitution,
the names and addresses of its officers, and the charter certificate
issued by the national union FFW.
However, respondent never submitted separate by-laws, nor does it
appear that respondent ever intended to prepare a set thereof. Section
1(c), Rule VI, Book V of Department Order No. 9 provides that the
submission of both a constitution and a set of by-laws is required, or at
least an indication that the local/chapter is adopting the constitution
and by-laws of the federation or national union. A literal reading of the
provision might indicate that the failure to submit a specific set of bylaws is fatal to the recognition of the local/chapter. However, a critical
examination of respondents constitution reveals that it is sufficiently
comprehensive in establishing the necessary rules for its operation.
These premises considered, there is clearly no need for a separate set
of by-laws to be submitted by respondent.
SAN MIGUEL FOODS, Inc. VS. LAGUESMA, 263 S 69
FACTS: Petitioner union filed before DOLE a Petition for Direct
Certification or Certification Election among the supervisors and
exempt employees of the SMC Magnolia Poultry Products Plants of
Cabuyao, San Fernando and Otis.
Med-Arbiter Danilo L. Reynante issued an Order ordering the conduct
of certification election among the abovementioned employees of the
different plants as one bargaining unit.
San Miguel Corporation filed a Notice of Appeal with Memorandum on
Appeal, pointing out, among others, the Med-Arbiters error in grouping
together all three (3) separate plants, into one bargaining unit, and in
including supervisory levels 3 and above whose positions are
confidential in nature.
The public respondent, Undersecretary Laguesma, granted respondent
companys Appeal and ordered the remand of the case to the MedArbiter of origin for determination of the true classification of each of
the employees sought to be included in the appropriate bargaining unit.
Upon petitioner-unions motion, Undersecretary Laguesma granted the
reconsideration prayed for and directed the conduct of separate
certification elections among the supervisors ranked as supervisory

SMC MANDAUE PACKING PRODUCTS VS. MANDAUE PACKING


PRODUCTS, GR NO. 152356, AUGUST 16, 2005
FACTS: On 15 June 1998, respondent, identifying itself as an affiliate
of Federation of Free Workers (FFW), filed a petition for certification
election with the DOLE Regional Office. In the petition, respondent
stated that it sought to be certified and to represent the permanent
rank-and-file monthly paid employees of the petitioner. A set of
documents were attached to the petition, including a (1) Charter
Certificate issued by FFW certifying that respondent was a duly
certified local chapter of FFW, (2) copy of the constitution of
respondent, (3) a list of respondents officers, (4) a certification
signifying that respondent had just been organized and no amount had
yet been collected from its members and (5) a list of all the rank-andfile monthly paid employees of the Mandaue Packaging Products
Plants and Mandaue Glass Plant.

15

levels 1 to 4 (S1 to S4) and the exempt employees in each of the three
plants at Cabuyao, San Fernando and Otis.
ISSUE: 1.Whether Supervisory employees 3 and 4 and the exempt
employees of the company are considered confidential employees,
hence ineligible from joining a union.
2. If they are not confidential employees, do the employees of the three
plants constitute an appropriate single bargaining unit.
RULING: (1) On the first issue, this Court rules that said employees do
not fall within the term confidential employees who may be prohibited
from joining a union.
They are not qualified to be classified as managerial employees who,
under Article 245 of the Labor Code, are not eligible to join, assist or
form any labor organization. In the very same provision, they are not
allowed membership in a labor organization of the rank-and-file
employees but may join, assist or form separate labor organizations of
their own.
Confidential employees are those who (1) assist or act in a confidential
capacity, (2) to persons who formulate, determine, and effectuate
management policies in the field of labor relations. The two criteria are
cumulative, and both must be met if an employee is to be considered a
confidential employee that is, the confidential relationship must exist
between the employee and his supervisor, and the supervisor must
handle the prescribed responsibilities relating to labor relations.
The exclusion from bargaining units of employees who, in the normal
course of their duties, become aware of management policies relating
to labor relations is a principal objective sought to be accomplished by
the confidential employee rule. The broad rationale behind this rule is
that employees should not be placed in a position involving a potential
conflict of interests. Management should not be required to handle
labor relations matters through employees who are represented by the
union with which the company is required to deal and who in the
normal performance of their duties may obtain advance information of
the companys position with regard to contract negotiations, the
disposition of grievances, or other labor relations matters.
The Court held that if these managerial employees would belong to or
be affiliated with a Union, the latter might not be assured of their loyalty
to the Union in view of evident conflict of interest. The Union can also
become company-dominated with the presence of managerial
employees in Union membership.
An important element of the confidential employee rule is the
employees need to use labor relations information. Thus, in
determining the confidentiality of certain employees, a key question
frequently considered is the employees necessary access to
confidential labor relations information.
(2) The fact that the three plants are located in three different places,
namely, in Cabuyao, Laguna, in Otis, Pandacan, Metro Manila, and in
San Fernando, Pampanga is immaterial. Geographical location can be
completely disregarded if the communal or mutual interests of the
employees are not sacrificed.
An appropriate bargaining unit may be defined as a group of
employees of a given employer, comprised of all or less than all of the
entire body of employees, which the collective interest of all the
employees, consistent with equity to the employer, indicate to be best
suited to serve the reciprocal rights and duties of the parties under the
collective bargaining provisions of the law.
A unit to be appropriate must effect a grouping of employees who have
substantial, mutual interests in wages, hours, working conditions and
other subjects of collective bargaining.
ACEDERA VS. INTERNATIONAL
SERVICES, Inc. GR NO. 146073

CONTAINER

increase by 365days and dividing by12 months, ICTSI stopped using


304 days as divisor and started using 365 days to determine the daily
wage.
Later on, ICTSI entered into a retrenchment program which prompted
APCWU to file a complaint before the Labor Arbiter (LA) for ICTSIs
use of 365 days, instead of 250days, as divisor in the computation of
wages. Acedera et al. filed a Motion to Intervene which was denied by
the LA. On appeal, National Labor Relations Commission (NLRC)
affirmed LAs decision. Acedera et al. filed a petition for certiorari to the
Court of Appeals (CA) which was dismissed.
ISSUE: Whether or not Acedera et al. have no legal right to intervene
in the case as their intervention was a superfluity.
HELD: Acedera et al. stress that they have complied with the
requisites for intervention because (1) they are the ones who stand to
gain or lose by the direct legal operation and effect of any judgment
that may be rendered in this case, (2) no undue delay or prejudice
would result from their intervention since their Complaint-inIntervention with Motion for Intervention was filed while the Labor
Arbiter was still hearing the case and before any decision thereon was
rendered, and (3) it was not possible for them to file a separate case
as they would be guilty of forum shopping because the only forum
available for them was the Labor Arbiter.
Acedera et al., however, failed to consider, in addition to the rule on
intervention, the rule on representation. A labor union is one such party
authorized to represent its members under Article 242(a) of the Labor
Code which provides that a union may act as the representative of its
members for the purpose of CBA. This authority includes the power to
represent its members for the purpose of enforcing the provisions of
the CBA.
That APCWU acted in a representative capacity "for and in behalf of its
Union members and other employees similarly situated, the title of the
case filed by it at the Labor Arbiters Office so expressly states.
While a party acting in a representative capacity, such as a union, may
be permitted to intervene in a case, ordinarily, a person whose
interests are already represented will not be permitted to do the same
except when there is a suggestion of fraud or collusion or that the
representative will not act in good faith for the protection of all interests
represented by him.
Acedera et al. cite the dismissal of the case filed by ICTSI, first by the
Labor Arbiter, and later by the Court of Appeals. The dismissal of the
case does not, however, by itself show the existence of fraud or
collusion or a lack of good faith on the part of APCWU. There must be
clear and convincing evidence of fraud or collusion or lack of good faith
independently of the dismissal. This, Acedera et al. failed to proffer.
Acedera et al. likewise express their fear that APCWU would not
prosecute the case diligently because of its sweetheart relationship"
with ICTSI. There is nothing on record, however, to support this alleged
relationship which allegation surfaces as a mere afterthought because
it was never raised early on. It was raised only in petitionersappellants reply to ICTSIs comment in the petition at bar, the last
pleading submitted to this Court, which was filed on June 20, 2001 or
more than 42 months after petitioners.
Appellants filed their Complaint-in-Intervention with Motion to Intervene
with the Labor Arbiter. To reiterate, for a member of a class to be
permitted to intervene in a representative action, fraud or collusion or
lack of good faith on the part of the representative must be proven. It
must be based on facts borne on record. Mere assertions, as what
petitioners-appellants proffer, do not suffice. Petition is denied.

TERMINAL

FULL CASE: MINETTE BAPTISTA ET. AL VS. ROSARIO


VILLANUEVA AND 15 OTHERS, GR NO. 146073, JANUARY 13,
2013

FACTS: Ordinarily, a person whose interests are already represented


will not be permitted to do the same except when there is a suggestion
of fraud or collusion or that the representative will not act in good faith.
Jerry Acedera, et al. are employees of International Container Terminal
Services, Inc.(ICTSI) and are members of Associated Port Checkers &
Workers Union-International Container Terminal Services, Inc.
(APCWU-ICTSI), a duly registered labor organization. ICTSI entered
into a five-year Collective Bargaining Agreement (CBA) with APCWU
which reduced the employees work days from 304 to 250 days a year.
The Wage Board decreed wage increases in NCR which affected
ICTSI. Upon the request of APCWU to compute the actual monthly
increase in the employees salary by multiplying the mandated

This Petition for Review on Certiorari under Rule 45 of the 1997 Rules
of Civil Procedure filed by Minette Baptista, Bannie Edsel San Miguel
and Ma. Fe Dayon (petitioners) assails the March 9, 2010 Decision
and the December 1, 2010 Resolution of the Court of Appeals (CA) in
CA-G.R. SP No. 105027, which affirmed the March 31, 2008 Decision
of the National Labor Relations Commission (NLRC) dismissing the
complaint for Unfair Labor Practice (ULP) filed against the named
respondents.

15

The Facts
Petitioners were former union members of Radio Philippines Network
Employees Union (RPNEU), a legitimate labor organization and the
sole and exclusive bargaining agent of the rank and file employees of
Radio Philippines Network (RPN), a government-sequestered
corporation involved in commercial radio and television broadcasting
affairs, while the respondents were the unions elected officers and
members.
On April 26, 2005, on suspicion of union mismanagement, petitioners,
together with some other union members, filed a complaint for
impeachment of their union president, Reynato Siozon, before the
executive board of RPN, which was eventually abandoned. They later
re-lodged the impeachment complaint, this time, against all the union
officers and members of RPNEU before the Department of Labor and
Employment (DOLE). They likewise filed various petitions for audit
covering the period from 2000 to 2004.
Thereafter, two (2) written complaints, dated May 26, 2005 and May
27, 2005, were filed against petitioners and several others for alleged
violation of the unions Constitution and By-Laws. Months later, on
September 19, 2005, a different group of union members filed a third
complaint against petitioners and 12 others, before the Chairman of
RPNEUs Committee on Grievance and Investigation (the Committee)
citing as grounds the "commission of an act which violates RPNEU
Constitution and By-Laws, specifically, Article IX, Section 2.2 for joining
or forming a union outside the sixty (60) days period and Article IX,
Section 2.5 for urging or advocating that a member start an action in
any court of justice or external investigative body against the Union or
its officer without first exhausting all internal remedies open to him or
available in accordance with the CBL." These complaints were, later
on, consolidated.
Thereafter, petitioners received a memorandum notice from Jeric
Salinas, Chairman of the Committee, requesting them to answer the
complaint and attend a hearing scheduled on October 3, 2005.
Petitioners and their group, through an exchange of communications
with the Committee, denied the charges imputed against them and
contested the procedure adopted by the Committee in its investigation.
On November 9, 2005, the Committee submitted their recommendation
of expulsion from the union to RPNEUs Board of Directors. On
December 21, 2005, the RPNEUs Board of Directors affirmed the
recommendation of expulsion of petitioners and the 12 others from
union membership in a Board Resolution No. 018-2005. Through a
Memorandum, dated December 27, 2005, petitioners were served an
expulsion notice from the union, which was set to take effect on
December 29, 2005. On January 2, 2006, petitioners with the 12 others
wrote to RPNEUs President and Board of Directors that their expulsion
from the union was an ultra vires act because the Committee failed to
observe the basic elements of due process because they were not
given the chance to physically confront and examine their
complainants.
In a letter, dated January 24, 2006, RPNEUs officers informed their
company of the expulsion of petitioners and the 12 others from the
union and requested the management to serve them notices of
termination from employment in compliance with their CBAs union
security clause. On February 17, 2006, RPN HRD Manager, Lourdes
Angeles, informed petitioners and the 12 others of the termination of
their employment effective March 20, 2006, enforcing Article II, Section
2 also known as the union security clause of their current CBA.
Aggrieved, petitioners filed three (3) separate complaints for ULP
against the respondents, which were later consolidated, questioning
legality of their expulsion from the union and their subsequent
termination from employment.
In a decision, dated April 30, 2007, the Labor Arbiter (LA) ruled in favor
of the petitioners and adjudged the respondents guilty of ULP pursuant
to Article 249 (a) and (b) of the Labor Code. The LA clarified that only
the union officers of RPNEU could be held responsible for ULP, so they
exonerated six (6) of the original defendants who were mere union
members. The LA also ordered the reinstatement of petitioners as
bonafide members of RPNEU. The decretal portion reads:
WHEREFORE, premises above considered, a decision is being issued
declaring union officers Ruth Bayquen, Ruby Castaeda, Alfred
Landas, Roce Garces, Board of Directors Federico Muoz, Janette
Roldan, Rosario Villanueva, Menandro Samson, Salvador Diwa and
Eugene Cruz guilty of unfair labor practice for violating Article 249,
paragraph A and B of the Labor Code. Respondents are also ordered

to cease and desist from further committing unfair labor practice and
order the reinstatement of the complainants as bonafide members of
the union.
The other claims are hereby denied for lack of factual and legal basis.
SO ORDERED.
Undaunted, the respondents appealed the LA decision to the NLRC.
In its Decision, dated March 31, 2008, the NLRC vacated and set aside
the LA decision and dismissed the complaint for ULP for lack of merit.
The NLRC found that petitioners filed a suit calling for the
impeachment of the officers and members of the Executive Board of
RPNEU without first resorting to internal remedies available under its
own Constitution and By-Laws. The NLRC likewise decreed that the
LAs order of reinstatement was improper because the legality of the
membership expulsion was not raised in the proceedings and, hence,
beyond the jurisdiction of the LA. The fallo of the NLRC decision reads:
WHEREFORE, the partial appeal filed by the respondents is
GRANTED. The decision, dated 30 April 2007 is VACATED and SET
ASIDE. The complaint is dismissed for lack of merit. SO ORDERED.
Petitioners filed for a motion for reconsideration, but the NLRC denied
it in its Resolution, dated May 30, 2008.
The CA, in its March 9, 2010 Decision, sustained the NLRC decision.
The CA stated that the termination of employment by virtue of a union
security clause was recognized in our jurisdiction. It explained that the
said practice fortified the union and averted disunity in the bargaining
unit within the duration of the CBA. The CA declared that petitioners
were accorded due process before they were removed from office. In
fact, petitioners were given the opportunity to explain their case and
they actually availed of said opportunity by submitting letters containing
their arguments.
Petitioners moved for reconsideration, but the CA likewise denied the
same in its December 1, 2010 Resolution, The CA expounded:
Anent petitioners charge of ULP against respondents, the records are
barren of proof to sustain such charge. What remains apparent is that
petitioners were expelled from the union due to their violation of
Section 2.5 of Article IX of the CBL which punishes the act of "urging or
advocating that a member start an action in any court of justice or
external investigative body against the Union or any of its officer,
without first exhausting all internal remedies open to him or available in
accordance with the Constitution and By-Laws of Union." As
petitioners expulsion was pursuant to the unions CBL, We absolve
respondents of the charges of ULP absent any substantial evidence to
sustain it.
The importance of a unions constitution and bylaws cannot be
overemphasized. They embody a covenant between a union and its
members and constitute the fundamental law governing the members
rights and obligations. As such, the unions constitution and bylaws
should be upheld, as long as they are not contrary to law, good morals
or public policy. In Diamonon v. Department of Labor and Employment,
the High Court affirmed the validity and importance of the provision in
the CBL of exhaustion of administrative remedies, viz:
When the Constitution and by-laws of both unions dictated the remedy
for intra-union dispute, such as petitioners complaint against private
respondents for unauthorized or illegal disbursement of union funds,
this should be resorted to before recourse can be made to the
appropriate administrative or judicial body, not only to give the
grievance machinery or appeals body of the union the opportunity to
decide the matter by itself, but also to prevent unnecessary and
premature resort to administrative or judicial bodies. Thus, a party with
an administrative remedy must not merely initiate the prescribed
administrative procedure to obtain relief, but also pursue it to its
appropriate conclusion before seeking judicial intervention.
Thus, petitioners advance the following
GROUNDS/ARGUMENTS IN SUPPORT OF THE PETITION
1. WITH DUE RESPECT, THE HONORABLE COURT OF APPEALS
MISERABLY FAILED TO APPRECIATE THE REAL ISSUE IN THIS
CASE.
2. WITH DUE RESPECT, THE DECISION AND RESOLUTION
ARRIVED AT BY THE HONORABLE COURT OF APPEALS ARE NOT
IN ACCORD WITH LAW AND APPLICABLE JURISPRUDENCE,
THEREBY GRAVELY ABUSING ITS DISCRETION AMOUNTING TO
LACK OR IN EXCESS OF JURISDICTION.
Petitioners submit that the respondents committed ULP under Article
289 (a) and (b) of the Labor Code. They insist that they were denied

15

substantive and procedural due process of law when they were


expelled from the RPNEU.
The petition is bereft of merit.
The primary concept of ULP is embodied in Article 247 of the Labor
Code, which provides:
Article 247. Concept of unfair labor practice and procedure for
prosecution thereof.Unfair labor practices violate the constitutional
right of workers and employees to self-organization, are inimical to the
legitimate interests of both labor and management, including their right
to bargain collectively and otherwise deal with each other in an
atmosphere of freedom and mutual respect, disrupt industrial peace
and hinder the promotion of healthy and stable labor-management
relations.
In essence, ULP relates to the commission of acts that transgress the
workers right to organize. As specified in Articles 248 and 249 of the
Labor Code, the prohibited acts must necessarily relate to the workers'
right to self-organization and to the observance of a CBA. Absent the
said vital elements, the acts complained, although seemingly unjust,
would not constitute ULP.
In the case at bench, petitioners claim that the respondents, as union
officers, are guilty of ULP for violating paragraphs (a) and (b) of Article
249 of the Labor Code, to wit:
ART.
249.
UNFAIR
LABOR
PRACTICES
OF
LABOR
ORGANIZATIONS.- It shall be unfair labor practice for a labor
organization, its officers, agents or representatives:
(a) To restrain or coerce employees in the exercise of their
rights to self-organization. However, a labor organization
shall have the right to prescribe its own rules with respect to
the acquisition or retention of membership:
(b) (b) To cause or attempt to cause an employer to discriminate
against an employee, including discrimination against an
employee with respect to whom membership in such
organization has been denied or to terminate an employee
on any ground other than the usual terms and conditions
under which membership or continuation of membership is
made available to other members;
Petitioners posit that the procedure that should have been followed by
the respondents in resolving the charges against them was Article
XVII, Settlement of Internal Disputes of their Constitution and By-Laws,
specifically, Section 2 thereof, requiring members to put their grievance
in writing to be submitted to their union president, who shall strive to
have the parties settle their differences amicably. Petitioners maintain
that any form of grievance would be referred only to the committee
upon failure of the parties to settle amicably.
The Court is not persuaded.
Based on RPNEUs Constitution and By-Laws, the charges against
petitioners were not mere internal squabbles, but violations that
demand proper investigation because, if proven, would constitute
grounds for their expulsion from the union. As such, Article X,
Investigation Procedures and Appeal Process of RPNEUs Constitution
and By-Laws, which reads
SECTION 1. Charge against any member or officer of the Union shall
be submitted to the Board of Directors (BOD) in writing, which shall
refer the same, if necessary, to the committee on Grievance and
Investigation. The Committee shall hear any charge and subsequently,
forward its finding and recommendation to the BOD. The BOD has the
power to approve or nullify the recommendation of the Committee on
Grievance and Investigation based on the merit of the appeal.
was correctly applied under the circumstances.
Besides, any supposed procedural flaw in the proceedings before the
Committee was deemed cured when petitioners were given the
opportunity to be heard. Due process, as a constitutional precept, is
satisfied when a person was notified of the charge against him and
was given an opportunity to explain or defend himself. In administrative
proceedings, the filing of charges and giving reasonable opportunity for
the person so charged to answer the accusations against him
constitute the minimum requirements of due process. The essence of
due process is simply to be heard, or as applied to administrative
proceedings, an opportunity to explain ones side, or an opportunity to
seek a reconsideration of the action or ruling complained of. It cannot
be denied that petitioners were properly notified of the charges filed
against them and were equally afforded the opportunity to present their
side.

Next, petitioners point out that they were not given the opportunity to
personally face and confront their accusers, which were violative of
their right to examine the complainants and the supposed charges
against them.
Petitioners contention is without merit. Mere absence of a one-on one
confrontation between the petitioners and their complainants does not
automatically affect the validity of the proceedings before the
Committee. Not all cases necessitate a trial-type hearing. As in this
case, what is indispensable is that a party be given the right to explain
ones side, which was adequately afforded to the petitioners.
It is well-settled that workers and employers organizations shall have
the right to draw up their constitutions and rules to elect their
representatives in full freedom, to organize their administration and
activities and to formulate their programs. In this case, RPNEUs
Constitution and By-Laws expressly mandate that before a party is
allowed to seek the intervention of the court, it is a pre-condition that
he should have availed of all the internal remedies within the
organization. Petitioners were found to have violated the provisions of
the unions Constitution and By-Laws when they filed petitions for
impeachment against their union officers and for audit before the
DOLE without first exhausting all internal remedies available within
their organization. This act is a ground for expulsion from union
membership. Thus, petitioners expulsion from the union was not a
deliberate attempt to curtail or restrict their right to organize, but was
triggered by the commission of an act, expressly sanctioned by Section
2.5 of Article IX of the unions Constitution and By-Laws.
For a charge of ULP against a labor organization to prosper, the onus
probandi rests upon the party alleging it to prove or substantiate such
claims by the requisite quantum of evidence. In labor cases, as in other
administrative proceedings, substantial evidence or such relevant
evidence as a reasonable mind might accept as sufficient to support a
conclusion is required. Moreover, it is indubitable that all the prohibited
acts constituting unfair labor practice should materially relate to the
workers' right to self-organization.
Unfortunately, petitioners failed to discharge the burden required to
prove the charge of ULP against the respondents. Aside from their selfserving allegations, petitioners were not able to establish how they
were restrained or coerced by their union in a way that curtailed their
right to self-organization. The records likewise failed to sufficiently
show that the respondents unduly persuaded management into
discriminating against petitioners. other than to bring to its attention
their expulsion from the union, which in turn, resulted in the
implementation of their CBA' s union security clause. As earlier stated,
petitioners had the burden of adducing substantial evidence to support
its allegations of ULP, which burden they failed to discharge. In fact,
both the NLRC and the CA found that petitioners were unable to prove
their charge of ULP against the respondents.
It is axiomatic that absent any clear showing of abuse, arbitrariness or
capriciousness, the findings of fact by the NLRC, especially when
affirmed by the CA, as in this case, are binding and conclusive upon
the Court. Having found none, the Court finds no cogent reason to
deviate from the challenged decision.
WHEREFORE, the petition is DENIED. The March 9, 2010 Decision
and the December 1, 2010 Resolution of the Court of Appeals in CAG.R. SP No. 105027 are AFFIRMED.
EDEN GLADS ABARIA et. al vs. NLRC, METRO CEBU COMM.
HOSPITAL, GR NO. 14113, 187778, 187861, 196156, DEC, 2011
FACTS: The consolidated petitions before us involve the legality of
mass termination of hospital employees who participated in strike and
picketing activities.
In a letter addressed to Nava, Ernesto Canen, Jr., Jesusa Gerona,
Hannah Bongcaras, Emma Remocaldo, Catalina Alsado and Albina
Baz, Atty. Alforque suspended their union membership for serious
violation of the Constitution and By-Laws.
Upon the request of Atty. Alforque, MCCHI granted one-day union
leave with pay for 12 union members. The next day, several union
members led by Nava and her group launched a series of mass
actions such as wearing black and red armbands/headbands,
marching around the hospital premises and putting up placards,
posters and streamers. For their continued picketing activities despite
the said warning, more than 100 striking employees were dismissed.
Unfazed, the striking union members held more mass actions. With the

15

volatile situation adversely affecting hospital operations and the


condition of confined patients, MCCHI filed a petition for injunction. A
temporary restraining order (TRO) was issued.
Thereafter, several complaints for illegal dismissal and unfair labor
practice were filed by the terminated employees against MCCHI, Rev.
Iyoy, UCCP and members of the Board of Trustees of MCCHI.
Executive Labor Arbiter Reynoso A. Belarmino rendered his decision
dismissing the complaints for unfair labor practice. Complainants
appealed to the Commission, which affirmed the Labor Arbiter.
Complainants elevated the case to the Court of Appeals (CA) (Cebu
Station) via a petition for certiorari. CAs Eighth Division dismissed the
petition on the ground that out of 88 petitioners only 47 have signed
the certification against forum shopping.
In G.R. No. 196156, MCCHI/VCMC prayed for the annulment of
Resolution of the CA, for this Court to declare the dismissal of
respondents Yballe, et al. as valid and legal and to reinstate the
Resolution of the NLRC.
G.R. No. 187861 was consolidated with G.R. Nos. 154113 and 187778
as they involve similar factual circumstances and identical or related
issues. G.R. No. 196156 was later also consolidated with the aforesaid
cases.

Separation pay is made an alternative relief in lieu of reinstatement in


certain circumstances, like: (a) when reinstatement can no longer be
effected in view of the passage of a long period of time or because of
the realities of the situation; (b) reinstatement is inimical to the
employer's interest; (c) reinstatement is no longer feasible; (d)
reinstatement does not serve the best interests of the parties involved;
(e) the employer is prejudiced by the workers continued employment;
(f) facts that make execution unjust or inequitable have supervened; or
(g) strained relations between the employer and employee. DENIED
HERITAGE HOTEL MANILA vs. PINAG-ISANG GALING AT LAKAS
NG MGA MANGGAGAWA SA HERITAGE MANILA, GR No. 177024

ISSUE: Whether or not respondents are illegally dismissed?

DOCTRINES: The Labor Code and its implementing rules do not


require that the number of members appearing on the documents in
question should completely dovetail. For as long as the documents
and signatures are shown to be genuine and regular and the
constitution and by-laws democratically ratified, the union is deemed to
have complied with registration requirements. Labor laws are liberally
construed in favor of labor especially if doing so would affirm its
constitutionally guaranteed right to self-organization. The right of any
person to join an organization also includes the right to leave that
organization and join another one.

HELD: Court of Appeals decision is sustained.


Records of the NCMB and DOLE Region 7 confirmed that NAMAMCCH-NFL had not registered as a labor organization, having
submitted only its charter certificate as an affiliate or local chapter of
NFL. Not being a legitimate labor organization, NAMA-MCCH-NFL is
not entitled to those rights granted to a legitimate labor organization
under Art. 242
Aside from the registration requirement, it is only the labor organization
designated or selected by the majority of the employees in an
appropriate collective bargaining unit which is the exclusive
representative of the employees in such unit for the purpose of
collective bargaining, as provided in Art. 255.NAMA-MCCH-NFL is not
the labor organization certified or designated by the majority of the
rank-and-file hospital employees to represent them in the CBA
negotiations but the NFL, as evidenced by CBAs concluded in 1987,
1991 and 1994.
Even assuming that NAMA-MCCH-NFL had validly disaffiliated from its
mother union, NFL, it still did not possess the legal personality to enter
into CBA negotiations. A local union which is not independently
registered cannot, upon disaffiliation from the federation, exercise the
rights and privileges granted by law to legitimate labor organizations;
thus, it cannot file a petition for certification election. Besides, the NFL
as the mother union has the right to investigate members of its local
chapter under the federations Constitution and By-Laws, and if found
guilty to expel such members. MCCHI therefore cannot be faulted for
deferring action on the CBA proposal submitted by NAMA-MCCH-NFL
in view of the union leaderships conflict with the national federation.
We have held that the issue of disaffiliation is an intra-union dispute
which must be resolved in a different forum in an action at the instance
of either or both the federation and the local union or a rival labor
organization, not the employer.
The above provision makes a distinction between workers and union
officers who participate in an illegal strike: an ordinary striking worker
cannot be terminated for mere participation in an illegal strike. There
must be proof that he or she committed illegal acts during a strike. A
union officer, on the other hand, may be terminated from work when he
knowingly participates in an illegal strike, and like other workers, when
he commits an illegal act during a strike.
Considering their persistence in holding picketing activities despite the
declaration by the NCMB that their union was not duly registered as a
legitimate labor organization and the letter from NFLs legal counsel
informing that their acts constitute disloyalty to the national federation,
and their filing of the notice of strike and conducting a strike vote
notwithstanding that their union has no legal personality to negotiate
with MCCHI for collective bargaining purposes, there is no question
that NAMA-MCCH-NFL officers knowingly participated in the illegal
strike. The CA therefore did not err in ruling that the termination of
union officers Perla Nava, Catalina Alsado, Albina Baz, Hannah
Bongcaras, Ernesto Canen, Jesusa Gerona and Guillerma Remocaldo
was valid and justified.

FACTS: The Heritage Hotel Employees Union (HHE) was formed in


2000 by certain rank and file employees
of
herein
petitioner
Heritage Hotel Manila, to which the Department of Labor
and Employment-National Capital Region issued a certificate of
registration. HHE filed a petition for certification election which
petitioner opposed on the ground that HHE misrepresented itself to be
an independent union, when in fact it was a local chapter of the
National Union of Workers in Hotel and Restaurant and Allied
Industries (NUWHRAIN). It was also alleged that such omitted
disclosure was intentional because petitioners supervisors union was
already affiliated with it. Petitioner also filed a petition to cancel the
unions registration certificate. The Med-Arbiter nevertheless granted
HHEs petition for certification election.
Petitioner appealed to the Secretary of Labor but it was denied as well
as its motion for reconsideration, prompting petitioner to file a petition
for certiorari with the Court of Appeals. The CA issued a writ of
injunction against the holding of HHEs certification election until the
petition for cancellation of its registration shall have been resolved with
finality.
In 2003, another union, herein respondent Pinag-Isang Galing at Lakas
ng mga Manggagawa sa Heritage Manila (PIGLAS), was formed by
certain rank and file employees of petitioner at a meeting. PIGLAS was
issued its registration certificate by the DOLE-NCR in 2004. HHE later
on adopted a resolution for its dissolution and then filed a petition for
cancellation of its union registration.
PIGLAS filed a petition for certification election which petitioner
opposed alleging that the new unions officers and members were also
those who comprised the old union. Petitioner likewise alleged that
PIGLAS was formed to circumvent the CAs injunction earlier issued.
The Med-Arbiter nevertheless granted the petition for certification
election.
Petitioner filed a petition to cancel PIGLAS registration, claiming that
the documents submitted with the unions application for registration
bore the following false information:
(a) The List of Members showed that the PIGLAS union had 100 union
members;
(b) The Organizational Minutes said that 90 employees attended the
meeting on December 10, 2003;
(c) The Attendance Sheet of the meeting of December 10, 2003 bore
the signature of 127 members who ratified the unions Constitution and
By-Laws; and
(d) The Signature Sheet bore 128 signatures of those who attended
that meeting.
Petitioner alleged that the misrepresentation was evidenced by the
discrepancy in the number of union members appearing in the
application and the list as well as the number of signatories to the
attendance and signature sheets. The minutes reported only 90
employees attended the meeting. Petitioner also alleged that 33
members of PIGLAS were members of HHE, which is in violation of the
policy against dual unionism.

15

DOLE-NCR denied petitioners petition to cancel PIGLAS


registration because the discrepancies in the number of members in
the applications supporting documents were not material and did not
constitute misrepresentation. The dual unionism is also not a ground
for canceling registration, since the members of HHE simply exercised
their right to self-organization and freedom of association when they
joined PIGLAS. The Bureau of Labor Relations affirmed the DOLENCR, by reasoning that PIGLAS organization meeting lasted for 12
hours. Thus, it was possible for the number of attendees to have
increased as the meeting progressed. Besides, the union only needed
50 members of the total of 250employees in the bargaining unit to
comply with the 20% membership requirement. Petitioner filed a
petition for certiorari with the Court of Appeals but it was denied as well
as petitioners motion for reconsideration. Hence, this petition for
review under Rule 45.

which prompted petitioners to appeal the decision to the director of the


Bureau of Labor Relations. The director of the BLR inhibited from the
issue, as he was previously the counsel of respondents. The Secretary
of Labor resolved the issue in the stead of the BLR director. She ruled
in favor of respondents. The petitioner filed a motion for
reconsideration of the decision, but was turned down. Petitioner then
filed for certiorari, challenging the jurisdiction of the DOLE Secretary.
An appeal from the decision of the Regional Director is supposed to be
under the jurisdiction of the BLR. Also, petitioner claims to have been
deprived of due process as it was not informed of the inhibition of the
BLR director.
ISSUES: Whether or not the ruling of the secretary of labor was valid.
Whether or not the petitioner was deprived of due process.
HELD: Petition is without merit.
Labor Law: It is without question that the appeal from the decision of
the regional office is within the jurisdiction of the BLR. Given the
circumstances, the BLR director inhibited himself. Petitioner insists that
the case should have gone to the subordinates of the BLR director.
However, this happens in cases where the director is incapacitated.
This does not obtain as the director merely inhibited himself. On the
other hand, the Secretary of DOLE has powers of supervision and
control over the BLR. As such, it may validly step into the shoes of the
BLR director and resolve the issue.

ISSUES: (1) Whether


or not
the
union made
fatal
misrepresentation in its application for union registration
(2) Whether or not dual unionism is a ground for canceling a unions
registration.
HELD: (1) NO.
Petitioner has no evidence of the alleged
misrepresentation. The discrepancies alone cannot be taken as
indication that PIGLAS misrepresented the information contained in
these documents. Charges of fraud and misrepresentation should be
clearly established by evidence and surrounding circumstances
because once it is proved, the labor union acquires none of the rights
accorded to registered organizations.
The discrepancies can be explained. While it appears that in the
minutes of the December 10,2003 organizational meeting, only 90
employees responded to the roll call at the beginning, it cannot be
assumed that such number could not grow to 128 as reflected on the
signature sheet for attendance. The meeting lasted 12 hours from
11:00am to 11:00pm. There is no evidence that the meeting hall was
locked up to exclude late attendees. As to the fact that only 127
members ratified the unions constitution and by-laws when 128 signed
the attendance sheet, it cannot be assumed that all those who
attended approved of such. Any member had the right to hold out and
refrain from ratifying those documents or to simply ignore the process.
The Labor Code and its implementing rules do not require that
the number of members appearing on the documents in question
should completely dovetail. For as long as the documents and
signatures are shown to be genuine and regular and the
constitution and by-laws democratically ratified, the union is
deemed to have complied with registration requirements.
The discrepancy in the list of members (showing only 100 members)
and the signature and attendance sheets (showing 127 or 128
members) submitted is immaterial. A comparison of the
documents shows that except for six members, the names found in the
list are also in the attendance and signature sheets. PIGLAS more
than complied with the 20% requirement since only 50 employees out
of250 employees in the bargaining unit were required to unionize.
Labor laws are liberally construed in favor of labor especially if
doing so would affirm its constitutionally guaranteed right to selforganization.
PIGLAS unions supporting documents reveal the
unmistakable yearning of petitioner companys rank and file employees
to organize. This yearning should not be frustrated by inconsequential
technicalities.
(2) NO. The right of any person to join an organization also
includes the right to leave that organization and join another one.
HHE union ceased to exist, its certificate of registration being already
cancelled. Thus, the petition is denied.

FULL CASE: TAKATA CORPORATION VS. BLR AND SALAMAT GR


No. 19676, JANUARY 12, 2011
FACTS: On July 7, 2009, petitioner filed with the Department of Labor
and Employment (DOLE) Regional Office a Petition3 for Cancellation
of the Certificate of Union Registration of Respondent Samahang
Lakas Manggagawa ng Takata (SALAMA1) on the ground that the
latter is guilty of misrepresentation, false statement and fraud with
respect to the number of those who participated in the organizational
meeting, the adoption and ratification of its Constitution and By-Laws,
and in the election of its officers. It contended that in the May 1, 2009
organizational meeting of respondent, only 68 attendees signed the
attendance sheet, and which number comprised only 17% of the total
number of the 396 regular rank- and-file employees which respondent
sought to represent, and hence, respondent failed to comply with the
20% minimum membership requirement. Petitioner insisted that the
document "Pangalan ng mga Kasapi ng Unyon" bore no signatures of
the alleged 119 union members; and that employees were not given
sufficient information on the documents they signed; that the document
"Sama-Samang Pahayag ng Pagsapi" was not submitted at the time of
the filing of respondent's application for union registration; that the 119
union members were actually only 117; and, that the total number of
petitioner's employees as of May 1, 2009 was 470, and not 396 as
respondent claimed.
Respondent denied the charge and claimed that the 119 union
members were more than the 20% requirement for union registration.
The document "Sama-Samang Pahayag ng Pagsapi sa Unyon" which
it presented in its petition for certification election supported their claim
of 119 members. Respondent also contended that petitioner was
estopped from assailing its legal personality as it agreed to a
certification election and actively participated in the pre-election
conference of the certification election proceedings. Respondent
argued that the union members were informed of the contents of the
documents they signed and that the 68 attendees to the organizational
meeting constituted more than 50% of the total union membership,
hence, a quo rumexisted for the conduct of the said meeting.
On August 27, 2009, DOLE Regional Director, Atty. Ricardo S.
Martinez, Sr., issued a Decision granting the petition for cancellation of
respondent's certificate of registration, the dispositive portion of which
reads:
WHEREFORE, from the foregoing considerations, the petition is
hereby GRANTED. Accordingly, the respondent Union Certificate of
Registration No. RO400A-2009-05-01-UR-LAG, dated May 19, 2009 is
hereby REVOCKED (sic) and /or CANCELLED pursuant to paragraph
(a) & (b), Section 3, Rule XIV of Department Order No. 40-03 and the
Samahang Lakas ng Manggagawa ng TAKATA (SALAMAT) is hereby
delisted from the roll of legitimate labor organization of this office.

HERITAGE HOTEL MANILA vs. NUWHRAIN-HHMSC, GR No.


178296, JANUARY 12, 2011
FACTS: Respondents filed a petition for certification of pre-election
with the DOLE. The Med-Arbiter approved the pre-election. However,
the certification election was delayed, but pushed through nonetheless.
Petitioner filed for cancellation of the certification due to the failure of
respondent to submit its financial statements to the Bureau of Labor
Relations. The Med-Arbiter still ruled in favor of respondents. Petitioner
appealed the decision to the regional director of the DOLE. The
Regional director still rendered a decision in favor of respondents,

15

In revoking respondent's certificate of registration, the Regional


Director found that the 68 employees who attended the organizational
meeting was obviously less than 20% of the total number of 396
regular rank-and-file employees which respondent sought to represent,
hence, short of the union registration requirement; that the attendance
sheet which contained the signatures and names of the union
members totaling to 68 contradicted the list of names stated in the
document denominated as "Pangalan ng mga Kasaping Unyon." The
document "Sama-Samang Pahayag ng Pagsapi" was not attached to
the application for registration as it was only submitted in the petition
for certification election filed by respondent at a later date. The
Regional Director also found that the proceedings in the cancellation of
registration and certification elections are two different and entirely
separate and independent proceedings which were not dependent on
each other.
Dissatisfied, respondent, through Bukluran ng Manggagawang Pilipino
(BMP) Paralegal Officer, Domingo P. Mole, filed a Notice and
Memorandum of Appeal with the Bureau of Labor Relations (BLR).
However, on September 28,2009, respondent, through its counsels,
Attys.
Napoleon C. Banzuela, Jr. and Jehn Louie W. Velandrez, filed an
Appeal Memorandum with Formal Entry of Appearance to the Office of
the DOLE Secretary, which the latter eventually referred to the BLR.
Petitioner filed an Opposition to the Appeals praying for their dismissal
on the ground of forum shopping as respondent filed two separate
appeals in two separate venues; and for failing to avail of the correct
remedy within the period; and that the certificate of registration was
tainted with fraud, misrepresentation and falsification.
In its Answer, respondent claimed that there was no forum shopping as
BMP's Paralegal Officer was no longer authorized to file an appeal on
behalf of respondent as the latter's link with BMP was already
terminated and only the Union President was authorized to file the
appeal; and that it complied with Department Order No. 40-03.
On December 9, 2009, after considering respondent's Appeal
Memorandum with Formal Entry of Appearance and petitioner's
Answer, the BLR rendered its Decision reversing the Order of the
Regional Director, the decretal portion of which reads:
WHEREFORE, the appeal is hereby GRANTED. The Decision of
Regional Director Ricardo S. Martinez, Sr., dated 27 August 2009, is
hereby REVERSED and SET ASIDE.
Accordingly, Samahang Lakas Manggagawa ng TAKATA (SALAMAT)
shall remain in the roster of labor organizations.
In reversing, the BLR found that petitioner failed to prove that
respondent deliberately and maliciously misrepresented the number of
rank-and-file employees. It pointed out petitioner's basis for the alleged
noncompliance with the minimum membership requirement for
registration was the attendance of 68 members to the May 1, 2009
organizational meeting supposedly comprising only 17% of the total
396 regular rank-and-file employees. However, the BLR found that the
list of employees who participated in the organizational meeting was a
separate and distinct requirement from the list of the names of
members comprising at least 20% of the employees in the bargaining
unit; and that there was no requirement for signatures opposite the
names of the union members; and there was no evidence showing that
the employees assailed their inclusion in the list of union members.
Petitioner filed a motion for reconsideration, which was denied by the
BLR in a Resolution dated January 8, 2010.
Undaunted, petitioner went to the CA via a petition for certiorari under
Rule 65.
After the submission of the parties' respective pleadings, the case was
submitted for decision.
On December 22, 2010, the CA rendered its assailed decision which
denied the petition and affirmed the decision of the BLR. Petitioner's
motion for reconsideration was denied in a Resolution dated March 29,
2011.
Hence this petition for review filed by petitioner raising the following
issues, to wit:
THE HONORABLE COURT OF APPEALS COMMITTED GRAVE AND
SERIOUS ERROR IN AFFIRMING THE DECISION OF PUBLIC
RESPONDENT BLR AND NOT FINDING ANY VIOLATION BY
SAMAHANG LAKAS MANGGAGAWA SA TAKATA (SALAMAT) OF
THE RULE ON FORUM SHOPPING IN THE FILING OF TWO
VERIFIED APPEALS FOR AND ITS BEHALF. BOTH OF THE

APPEALS SHOULD HAVE BEEN DISMISSED OUTRIGHT BY


PUBLIC RESPONDENT BLR, ON GROUND OF FORUM SHOPPING.
THE HONORABLE COURT OF APPEALS SERIOUSLY ERRED IN
FINDING THAT THE APPLICATION FOR REGISTRATION OF
SAMAHANG LAKAS MANGGAGAWA SA TAKATA (SALAMAT) WAS
COMPLIANT
WITH
THE
LAW.
CONSIDERING
THE
CIRCUMSTANCES OBTAINING IN THE REGISTRATION OF
SALAMAT, IT IS CLEAR THAT THE SAME IS TAINTED WITH FRAUD,
MISREPRESENTATION AND FALSIFICATION. SALAMAT DID NOT
POSSESS THE REQUIREDNUMBER OF MEMBERS AT THE TIME
OF FILING OF ITS APPLICATION FOR REGISTRATION, HENCE, IT
SHOULD BE HELD GUILTY OF MISREPRESENTATION, AND FALSE
STATEMENTS AND FRAUD IN CONNECTION THEREWITH.
Anent the first issue, petitioner contends that respondent had filed two
separate appeals with two different representations at two different
venues, in violation of the rule on multiplicity of suits and forum
shopping, and instead of dismissing both appeals, the appeal
erroneously filed before the Labor Secretary was the one held validly
filed, entertained and even granted; that it is not within the discretion of
BLR to choose which between the two appeals should be entertained,
as it is the fact of the filing of the two appeals that is being prohibited
and not who among the representatives therein possessed the
authority.
We are not persuaded.
We find no error committed by the CA in finding that respondent
committed no forum shopping. As the CA correctly concluded, to wit:
It is undisputed that BMP Paralegal Officer Domingo P. Mole was no
longer authorized to file an appeal on behalf of union SALAMAT and
that BMP was duly informed that its services was already terminated.
SALAMAT even submitted before the BLR its "Resolusyon Blg. 012009" terminating the services of BMP and revoking the representation
of Mr. Domingo Mole in any of the pending cases being handled by him
on behalf of the union. So, considering that BMP Paralegal Officer
Domingo P. Mole was no longer authorized to file an appeal when it
filed the Notice and Memorandum of Appeal to DOLE Regional Office
No. IV-A, the same can no longer be treated as an appeal filed by
union SALAMAT. Hence, there is no forum shopping to speak of in this
case as only the Appeal Memorandum with Formal Entry of
Appearance filed by Atty. Napoleon C. Banzuela, Jr. and Atty. Jehn
Louie W. Velandrez is sanctioned by SALAMAT.
Since Mole's appeal filed with the BLR was not specifically authorized
by respondent, such appeal is considered to have not been filed at all.
It has been held that "if a complaint is filed for and in behalf of the
plaintiff who is not authorized to do so, the complaint is not deemed
filed.
An unauthorized complaint does not produce any legal effect."
Respondent through its authorized representative filed its Appeal
Memorandum with Formal Entry of Appearance before the Labor
Secretary, and not with the BLR. As the appeal emanated from the
petition for cancellation of certificate of registration filed with the
Regional Office, the decision canceling the registration is appealable to
the BLR, and not with the Labor Secretary. However, since the Labor
Secretary motu propio referred the appeal with the BLR, the latter can
now act on it. Considering that Mole's appeal with the BLR was not
deemed filed, respondents appeal, through Banzuela and Associates,
which the Labor Secretary referred to the BLR was the only existing
appeal with the BLR for resolution. There is, therefore, no merit to
petitioner's claim that BLR chose the appeal of Banzuela and
Associates over Mole's appeal.
The case of Abbott Laboratories Philippines, Inc. v. Abbott Laboratories
Employees Union cited by petitioner is not at all applicable in this case
as the issue therein is the authority of the Labor Secretary to review
the decision of the Bureau of Labor Relations rendered in the exercise
of its appellate jurisdiction over decision of the Regional Director in
cases involving cancellations of certificate of registration of labor
unions. We found no grave abuse of discretion committed by the
Secretary of Labor in not acting on therein petitioner's appeal. The
decision of the Bureau of Labor Relations on cases brought before it
on appeal from the Regional Director are final and executory. Hence,
the remedy of the aggrieved party is to seasonably avail of the special
civil action of certiorari under Rule 65 and the Rules of Court. In this
case, after the Labor Secretary motu propio referred respondent's
appeal filed with it to the BLR which rendered its decision reversing the

15

Regional Director, petitioner went directly to the CA via a petition for


certiorari under Rule 65.
As to the second issue, petitioner seeks the cancellation of
respondent's registration on grounds offraud and misrepresentation
bearing on the minimum requirement of the law as to its membership,
considering the big disparity in numbers, between the organizational
meeting and the list of members, and so misleading the BLR that it
obtained the minimum required number of employees for purposes of
organization and registration.
We find no merit in the arguments.
Art. 234 of the Labor Code provides:
ART. 234. Requirements of Registration. - A federation, national union
or industry or trade union center or an independent union shall acquire
legal personality and shall be entitled to the rights and privileges
granted by law to legitimate labor organizations upon issuance of the
certificate of registration based on the following requirements:
(a) Fifty pesos (P50.00)registration fee;
(b) The names of its officers, their addresses, the principal
address of the labor organization, the minutes of the
organizational meetings and the list of the workers who
participated in such meetings;
(c) In case the applicant is an independent union, the names of
all its members comprising at least twenty percent (20%) of
all the employees in the bargaining unit where it seeks to
operate;
(d) If the applicant union has been in existence for one or more
years, copies of its annual financial reports; and
(e) Four copies of the constitution and by-laws of the applicant
union, minutes of its adoption or ratification, and the list of
the members who participated in it."
And after the issuance of the certificate of registration, the labor
organization's registration could be assailed directly through
cancellation of registration proceedings in accordance with Articles 238
and 239 of the Labor Code. And the cancellation of union certificate of
registration and the grounds thereof are as follows:
ART. 238. Cancellation of Registration. - The certificate of registration
of any legitimate labor organization, whether national or local, may be
cancelled by the Bureau, after due hearing, only on the grounds
specified in Article 239 hereof.
ART. 239. Grounds for Cancellation of Union Registration. - The
following may constitute grounds for cancellation of union registration:
(a) Misrepresentation, false statement or fraud in connection
with the adoption or ratification of the constitution and bylaws or amendments thereto, the minutes of ratification, and
the list of members who took part in the ratification;
(b) Misrepresentation, false statements or fraud in connection
with the election of officers, minutes of the election of
officers, and the list of voters;
(c) Voluntary dissolution by the members.
Petitioner's charge that respondent committed misrepresentation and
fraud in securing its certificate of registration is a serious charge and
must be carefully evaluated. Allegations thereof should be
compounded with supporting circumstances and evidence. We find no
evidence on record to support petitioner's accusation.
Petitioner's allegation of misrepresentation and fraud is based on its
claim that during the organizational meeting on May 1, 2009, only 68
employees attended, while respondent claimed that it has 119
members as shown in the document denominated as "Pangalan ng
mga Kasapi ng Unyon;" hence, respondent misrepresented on the
20% requirement of the law as to its membership.
We do not agree.
It does not appear in Article 234 (b) of the Labor Code that the
attendees in the organizational meeting must comprise 20% of the
employees in the bargaining unit. In fact, even the Implementing Rules
and Regulations of the Labor Code does not so provide. It is only
under Article 234 (c) that requires the names of all its members
comprising at least twenty percent (20%) of all the employees in the
bargaining unit where it seeks to operate. Clearly, the 20% minimum
requirement pertains to the employees membership in the union and
not to the list of workers who participated in the organizational meeting.
Indeed, Article 234 (b) and (c) provide for separate requirements,
which must be submitted for the union's registration, and which
respondent did submit. Here, the total number of employees in the
bargaining unit was 396, and 20% of which was about 79. Respondent

submitted a document entitled "Pangalan ng Mga Kasapi ng Unyon"


showing the names of 119 employees as union members, thus
respondent sufficiently complied even beyond the 20% minimum
membership requirement. Respondent also submitted the attendance
sheet of the organizational meeting which contained the names and
signatures of the 68 union members who attended the meeting.
Considering that there are 119 union members which are more than
20% of all the employees of the bargaining unit, and since the law
does not provide for the required number of members to attend the
organizational meeting, the 68 attendees which comprised at least the
majority of the 119 union members would already constitute a quorum
for the meeting to proceed and to validly ratify the Constitution and Bylaws of the union. There is, therefore, no basis for petitioner to contend
that grounds exist for the cancellation of respondent's union
registration. For fraud and misrepresentation to be grounds for
cancellation of union registration under Article 239 of the Labor Code,
the nature of the fraud and misrepresentation must be grave and
compelling enough to vitiate the consent of a majority of union
members.
Petitioner's claim that the alleged union members signed documents
without adequate information is not persuasive. The one who alleges a
fact has the burden of proving it and a mere allegation is not
evidence.23 In fact, we note that not one of those listed in the
document denominated as "Pangalan ng Mga Kasaping Unyon" had
come forward to deny their membership with respondent. Notably, it
had not been rebutted that the same union members had signed the
document entitled "Sama-Samang Pahayag ng Pagsapi," thus,
strengtheningtheir desire to be members of the respondent union.
Petitioner claims that in the list of members, there was an employee
whose name appeared twice and another employee who was merely a
project employee. Such could not be considered a misrepresentation in
the absence of showing that respondent deliberately did so for the
purpose of increasing their union membership. In fact, even if those
two names were not included in the list of union members, there would
still be 117 members which was still more than 20% of the 396 rankand-file employees.
As to petitioner's argument that the total number of its employees as of
May 1, 2009 was 470, and not396 as respondent claimed, still the 117
union members comprised more than the 20% membership
requirement for respondent's registration.
In Mariwasa Siam Ceramics v. Secretary of the Department of Labor
and Employment, we said:
For the purpose of de-certifying a union such as respondent, it must be
shown that there was misrepresentation, false statement or fraud in
connection with the adoption or ratification of the constitution and bylaws or amendments thereto, the minutes of ratification; or, in
connection with the election of officers, the minutes of the election of
officers, the list of voters, or failure to submit these documents together
with the list of the newly elected-appointed officers and their postal
addresses to the BLR.
The bare fact that two signatures appeared twice on the list of those
who participated in the organizational meeting would not, to our mind,
provide a valid reason to cancel respondents certificate of registration.
The cancellation of a unions registration doubtless has an impairing
dimension on the right of labor to self-organization. For fraud and
misrepresentation to be grounds for cancellation of union registration
under the Labor Code, the nature of the fraud and misrepresentation
must be grave and compelling enough to vitiate the consent of a
majority of union members.1wphi1
In this case, we agree with the BLR and the CA that respondent could
not have possibly committed misrepresentation, fraud, or false
statements. The alleged failure of respondent to indicate with
mathematical precision the total number of employees in the
bargaining unit is of no moment, especially as it was able to comply
with the 20% minimum membership requirement. Even if the total
number of rank-and-file employees of petitioner is 528, while
respondent declared that it should only be 455, it still cannot be denied
that the latter would have more than complied with the registration
requirement.
WHEREFORE, premises considered, the petition for review is
DENIED. The Decision dated December 22, 2010 and the Resolution
dated March 28, 2011 of the Court of Appeals, in CA-G.R. SP No.
112406, are AFFIRMED.

15

UST FACULTY UNION vs. BITONIO, GR No. 131235

such benefit because it shall be given only as a matter of discretion of


the employer.
More specifically, it is given only in consideration for the goodwill that
existed in the negotiations, which culminate in the signing of a CBA.
UST claims that since this condition (existence of CBA) is absent in the
parties case, it was erroneous to have rewarded respondent with an
increased signing bonus.

FACTS: Private respondent Mario et al are duly elected officer of UST


faculty. The union has a 5-year CBA with its employer and is set to
expire on May 31,1998. On September 21, 1996, Sec Gen of the union
posted a general assembly announcement to be held on October 5,
1996. Various UST club presidents requested a general faculty
assembly thus union and non-union faculty members convened. New
set of officers were elected, violative of the CBL and that GA was held
with the attendance of non-union members. Current union officers
were served with a notice to vacate the union office as new set of
offices were already elected. CBA was likewise ratified by an
overwhelming majority. Mad-Arbiter declared the election conducted
was violative of the unions CBL. BLR Director Bitonio upheld the
decision with a ruling that the CBL which constituted the covenant
between the union and its members, could not be suspended during
the general assembly of all faculty members, since it had not been
authorized by the union.

Issue # 1: Whether or not the members of private respondent


voluntarily and knowingly accepted the arbitral award of the secretary
of dole, amounting to ratification or waiver. (No)
The union members
individual acceptance
of rewards do not
operate as a ratification of the DOLE Secretarys award; nor a waiver
of their right to receive further benefits, or what they may be entitled to
under the law.
The appellate court correctly ruled that the respondents members
were merely constrained to accept payment at the time.
Christmas was then just around the corner, and the union members
were in no position to resist the temptation to accept much-needed
cash for use during the most auspicious occasion of the year.
The SC pointed out that as individual components of a union
possessed of a distinct and separate corporate personality, members
of the union should realize that in joining the organization, they have
surrendered a portion of their individual freedom for the benefit of all
the other members; they submit to the will of the majority of the
members in order that they may derive the advantages to be gained
from the concerted action of all.
Since the will of the members is personified by its board of directors or
trustees, the decisions it makes should accordingly bind them.
A labor union exists in whole or in part for the purpose of
collective
bargaining or of
dealing with employers concerning terms and
conditions of employment.
What the individual employee may not do alone, as for example obtain
more favorable terms and conditions of work, the labor organization,
through persuasive and coercive power gained as a group, can
accomplish better.

ISSUE: Whether or not the public respondent committed grave abuse


of discretion in refusing to recognize the officers elected during the
general assembly.
HELD: Self-organization is a fundamental right guaranteed by the
constitution and labor Code. Corollary to this right is the prerogative
not to join, affiliate with or assist a labor union. Therefore, to become a
union member, an employee must not only signify the intent to become
one but also take some positive steps to realize that intent. The
procedure for union membership is usually embodied in the unions
CBL. An employee who becomes a union member acquires the rights
and the concomitant obligations that go with the new status and
becomes bound by the unions rules and regulations.
Union election hold pursuant to the unions CBL, and the right to vote
in it is enjoyed only by union members.
Certification election is the process of determining, thorough secret
ballot, the sole and exclusive bargaining agent of the employees in the
appropriate bargaining unit for the purpose of collective bargaining the
purpose to ascertain whether or not a majority of the employees wish
to be represented by a labor organization and by which particular labor
organization.

Issue # 2: Whether or not the increase of the signing bonus was


correct. (No)
A signing bonus is a grant motivated by the goodwill generated when a
CBA is successfully negotiated and signed between the employer and
the union.
In the case at bar, no CBA was successfully negotiated by the parties.
However, the SC will still allow an award of signing bonus because
UST prayed to affirm in toto the DOLE Secretarys Order (which
included the grant of Php 10,000.00 signing bonus).
In seeking to affirm the DOLE Secretarys Order, UST agreed to the
grant of the original award of Php10,000.00 and not to Php 18,000.00.
Thus, there is no basis for the increase.
Disposition: the petition is PARTIALLY GRANTED. The signing
bonus of EIGHTEEN THOUSAND PESOS(P18,000.00) per member
of respondent Samahang Manggagawa ng U.S.T. as awarded by
the Court of Appeals is REDUCED to TEN THOUSAND PESOS
(P10,000.00). All other findings and dispositions made by the Court of
are AFFIRMED.

UST vs. SAMAHANG MANGGAGAWA NG UST, GR No. 169940


FACTS: Respondent Samahang Manggagawa ng U.S.T. (SM-UST)
was the authorized bargaining agent of the non-academic/nonteaching rank-and-file daily- and monthly-paid employees
(numbering about 619) of UST.
Union & UST attempted to have a CBA formulated, containing salary
increases, signing and Christmas bonuses.
Basically the union was arguing for higher amounts of additional
benefits and claimed that such grants may be shouldered not solely
from what has been collected from the tuition fee increase, but also
from USTs other sources.
On the other hand, UST argued that what the union was asking for is
too much. It argued that as an educational institution, its main focus
shall be the quality of education it gives to its students. Hence, the
funds must be balanced and that the fact that not much of their funds
are allotted to their employees is justified.
In the end, the parties were not able to come to an agreement; thus,
the respondent union voted to stage a strike.
This was promptly acted upon by DOLE, which rendered a decision
granting, among others, reasonable increases and signing bonus
an order much in favour of UST.
SOLEs Order was then appealed to the CA which partially granted the
petition. CA granted an increase in thesigning bonus. Hence, this
petition where UST alleged the following:
It began paying the wage adjustment and other benefits pursuant to
the May 31, 2002 Order of the DOLE Secretary (572 out of the
619 members of respondent have been paid). It argued that
by their acceptance of the award and the resulting payments made to
them, the said union members have ratified its offer and thus rendered
moot the case before the Court of Appeals.
The CAs award of additional signing bonus (from P10,000.00 to
P18,000.00) is contrary to the nature and principle behind the grant of

VILLAR VS. INCIONG, 121 S 44


FACTS: AEU under FUR attempted to have a certification election but
due to the opposition of AEU-PAFLU, the petition was denied by the
Med-Arbiter.
AEU-PAFLU then called a special meeting among members and it was
there decided that an investigation of certain people would be held
pursuant to the constitution and by-laws of the Federation, of all of the
petitioners and one Felipe Manlapao, for "continuously maligning,
libelling and slandering not only the incumbent officers but even the
union itself and the federation;" spreading 'false propaganda' that the
union officers were 'merely appointees of the management', and for
causing divisiveness in the union.
A Trial Committee was then formed to investigate the local union's
charges against the petitioners for acts of disloyalty. AEU-PAFLU and
the Company concluded a new CBA which, besides granting additional
benefits to the workers, also reincorporated the same provisions of the
existing CBA, including the union security clause reading, to wit:

15

All members of the UNION as of the signing of this Agreement shall


remain members thereof in good standing. Therefore, any members
who shall resign, be expelled, or shall in any manner cease to be a
member of the UNION, shall be dismissed from his employment upon
written request of the UNION to the Company.
The petitioners were summoned to appear before the PAFLU Trial
Committee for the aforestated investigation of the charges filed against
them but they did not attend and instead requested for a "Bill of
Particulars" of the charges which had been formalized by the AEUPAFLU officers; they contend that their actions were merely exercise of
the right to freedom of association.
Not recognizing PAFLU's jurisdiction over their case, petitioners again
refused to participate in the investigation rescheduled and conducted.
Instead, they merely appeared to file their Answer to the charges and
moved for a dismissal.
Based on the findings and recommendations of the PAFLU trial
committee, the PAFLU President found the petitioners guilty of the
charges against them and it was requested that they be terminated in
conformity with the security clause in the CBA. Meanwhile, they were
placed under preventive suspension and denied access to the
workplace.
ISSUE: Whether or not the Minister acted with grave abuse of
discretion when he affirmed the decision of the RO4-Officer-in-Charge
allowing the preventive suspension and subsequent dismissal of
petitioners by reason of the exercise of their right to freedom of
association.
HELD: It is true that disaffiliation from a labor union is not open to legal
objection. It is implicit in the freedom of association ordained by the
Constitution. However, a closed shop is a valid form of union security,
and such provision in a CBA is not a restriction of the right of freedom
of association guaranteed by the Constitution.
Here, the Company and the AEU-PAFLU entered into a CBA with a
union security clause and the stipulation for closed-shop is clear and
unequivocal and it leaves no room for doubt that the employer is
bound, under the collective bargaining agreement, to dismiss the
employees, herein petitioners, for non-union membership.
Petitioners became non-union members upon their expulsion from the
general membership of the AEU-PAFLU pursuant to the Decision of
the PAFLU national president.
PAFLU had the authority to investigate petitioners on the charges filed
by their co-employees in the local union and after finding them guilty as
charged, to expel them from the roll of membership under the
constitution of the PAFLU to which the local union was affiliated.
According to the OIC: dtripped of non-essentials, the basic and
fundamental issue in this case tapers down to the determination of
WHETHER OR NOT PAFLU HAD THE AUTHORITY TO
INVESTIGATE OPPOSITORS AND, THEREAFTER, EXPEL THEM
FROM THE ROLL OF MEMBERSHIP OF THE AMIGOEMPLOYEES
UNION-PAFLU.
Recognized and salutary is the principle that when a labor union
affiliates with a mother union, it becomes bound by the laws and
regulations of the parent organization.
When a labor union affiliates with a parent organization or mother
union, or accepts a charter from a superior body, it becomes subject to
the laws of the superior body under whose authority the local union
functions. The constitution, by-laws and rules of the parent body,
together with the charter it issues pursuant thereto to the subordinate
union, constitute an enforceable contract between the parent body and
the subordinate union, and between the members of the subordinate
union inter se.
'Due process' simply means that the parties were given the opportunity
to be heard. In the instant case, ample and unmistakable evidence
exists to show that the oppositors were afforded the opportunity to
present their evidence, but they themselves disdained or spurned the
said opportunity given to them.
Inherent in every labor union, or any organization, is the right of selfpreservation. When members of a labor union, therefore, sow the
seeds of dissension and strife within the union; when they seek the
disintegration and destruction of the very union to which they belong,
they thereby forfeit their rights to remain as members of the union
which they seek to destroy.
We, therefore, hold and rule that petitioners, although entitled to
disaffiliate from their union and form a new organization of their own,

must, however, suffer the consequences of their separation from the


union under the security clause of the CBA.
SALUNGA vs. CIR, 21 S 216
FACTS: Petitioner Francisco Salunga was a member of the Union
since 1953. Due to a falling out with the Union, he tendered his
resignation from the Union, which accepted it and transmitted it to the
Company with a request for the immediate implementation of said
section 3. SMB informed Salunga that his aforementionedresignation
would result in the termination of his employment, and in view of said
section, Salunga wrote a letter to the Union withdrawing or
revoking his resignation and advising the Union to continue deducting
his monthly union dues. He, moreover, furnished a copy of this
communication to SMB. The latter, in turn, notified the Union of the
receipt of said copy and that "in view thereof, we shall not take any
action on this case and shall consider Mr. Francisco Salunga still a
member of your union and continue deducting his union dues."
However, on September 8, 1961, the Union told SMB that Salungas
membership could not be reinstated and insisted on his
separation
from
the
service, conformably with the stipulation
above-quoted. SMB sent a reply clarifying the instructions of the Union
to terminate the employment of Salunga. The Union reiterated its
request for implementation of said section 3, for which reason, SMB
notified Salunga that, in view of said letter and the aforementioned
section, they have to terminate his employment, although with regret.
Meanwhile, Salunga sought the intervention of PAFLU's National
President, Cipriano Cid, to which the Union was affiliated, for a review
of the latter's action. PAFLU gave due course to petitioner's request for
review.
On October 6, 1961, Cid advised Salunga that PAFLU had found no
ground to review the action taken by the Union and that, on the
expiration of the 15-daygrace granted to him by the Company, the
decision thereof to terminate his services would take effect.
Salunga then notified the PAFLU that he was appealing to its supreme
authority the PAFLU National Convention and requested that
action on his case be deferred until such time as the Convention
shall have
acted on his appeal. Furthermore, he asked SMB to
maintain the status quo, in the meantime. This notwithstanding, at the
close of the business hours, on October 15, 1961, Salunga was
discharged from the employment of the Company, through its
assistant-secretary and vice-president, herein respondent Miguel
Noel. Hence, the complaint in the CIR.
CIR decided in favor of Salunga. Defendants (including SMB) all guilty
of ULP. Ordered Union to readmit petitioner as member, and Company
to reinstate him with backwages.
ON Motion for Reconsideration, CIR reversed its earlier decision.
Hence, this appeal by the petitioner.
ISSUES/HELD: WON the Union is guilty of ULP. YES.
WON SMB (the employer) is guilty of ULP. NO
RATIO:
1.

In the case of the Union, it was found that NBAILUP-PAFLU


acted arbitrarily. Although Salunga had resigned from the
Union and the latter had accepted the resignation, Salunga
had, soon later upon learning that his withdrawal from the
Union would result in his separation from the Company,
owing to the closed-shop provision above referred to
revoked or withdrawn said resignation, and the Union
refused to consent thereto without any just cause therefor.
The Union had not only acted arbitrarily in not allowing
petitioner to continue his membership but the trial Judge also
found said refusal of the Union officers to be due to his
critical attitude towards certain measures taken or
sanctioned by them.

The record is clear that, feeling dejected by the inaction of the union
officials on his grievances and objections to what he believed were
illegal disbursements of union funds, coupled with the fact that he was
later removed from his position as a union steward without his
knowledge, as well as the fact that the union did not honor the power
of attorney executed in his favor by Alejandro Miranda, a co-worker,
for the collection of Miranda's indebtedness of P60.00 to

15

him, he submitted his letter of resignation from the union on August


18, 1961.
Indeed, said officers tried to justify themselves by characterizing said
criticisms as acts of disloyalty to the Union, which, of course, is not
true, not only because the criticism assailed, not the Union, but certain
acts of its officers, and, indirectly, the officers themselves, but also
because the constitution and by-laws of the Union explicitly recognize
the right of its members to give their views on "all
transactions made by the Union." As a consequence, the resolution
appealed from cannot be affirmed without, in effect, nullifying said right
which, independently of the constitution and by-laws of the Union, is
part and parcel of the freedom of speech guaranteed in the
Constitution of our Republic, as a condition sine qua non to the sound
growth and development of labor organizations and democratic
institutions.
Although, generally, a state may not compel ordinary voluntary
associations to admit thereto any given individual, because
membership therein may be accorded or withheld as a matter of
privilege, the rule is qualified in respect of labor unions holding a
monopoly in the supply of labor, either in a given locality, or as regards
particular employer with which it has a closed-shop agreement. The
reason is that:
The closed shop and the union shop cause the
admission requirements of trade union to become affected with
the public interest. Likewise, a closed shop, a union shop, or
maintenance of membership clauses cause the administration of
discipline by unions to be affected with the public interest.
Consequently, it is well settled that such unions are not entitled to
arbitrarily exclude qualified applicants for membership, and a closedshop provision would not justify the employer in discharging, or a union
in insisting upon the discharge of, an employee whom the union thus
refuses to admit to membership, without any reasonable ground
therefor.
Needless to say, if said unions may be compelled to admit new
members, who have the requisite qualifications, with more reason may
the law and the courts exercise the coercive power when the employee
involved is a long standing union member, who, owing to provocations
of union officers, was impelled to tender his resignation, which he
forthwith withdrew or revoked. Surely, he may, at least, invoke the
rights of those who seek admission for the first time, and cannot
arbitrarily be denied readmission.
2. For SMB, however, it was held that it did not engage in ULP.
It was shown that SMB itself was even reluctant if not
unwilling to discharge the petitioner.
When the Union first informed SMB of Salungas resignation
and urged implementation of section 3 of the bargaining contract,
SMB advised Salunga ofthe provision, thereby intimating that he had to
withdraw his resignation in order to keep his employment. Besides,
SMB notified the Union that it (SMB) would not take any action on the
case and would consider Salunga "still a member" of the Union. When
the latter, thereafter, insisted on petitioner's discharge, SMB still
demurred and explained it was not taking sides and that its stand was
prompted merely by "humane" considerations, springing from the belief
that petitioner had resigned from the Union without realizing its effect
upon his employment. And, as the Union reiterated its demand, SMB
notified petitioner that it had no other alternative but to terminate his
employment, and dismissed him from the service, although with
"regret".
Under these circumstances, SMB was not "unfair to the petitioner.
On the contrary, it did not merely show a commendable understanding
of and sympathy for his plight. It even tried to help him, although to
such extent only as was consistent with its obligation to refrain from
interfering in purely internal affairs of the Union. At the same time, SMB
could not safely inquire into the motives of the Union officers, in
refusing to allow Salunga to withdraw his resignation.
Inasmuch as the true motives were not manifest, without such
inquiry, and Salunga had concededly tendered his resignation of his
own free will, the arbitrary nature of the decision of said officers was
not such as to be apparent and to justify the company in regarding said
decision unreasonable. Upon the other hand, SMB cannot be blamed
for assuming the contrary, for Salunga had appealed to the National
Officers of the PAFLU and the latter had sustained the Union. SMB
was therefore justified in presuming that the PAFLU had inquired into
all relevant circumstances, including the motives of the Union Officers.

In finding the company guilty of ULP, the trial Judge felt that San
Miguel should have waited for the action of the national convention
before issuing the notice of dismissal. However, the record does not
show that petitioner was prejudiced by San Miguels failure to
maintain the status quo, after the Union had been sustained by
said officers. In fact, petitioner did not even try to establish that he had
submitted to San Miguel as he has not introduced in the lower court
satisfactory proof that an appeal had really been taken by
him to the aforementioned Convention.
Bugay v. Kapisanan ng Mga Manggagawa sa MRR, 4 SCRA 487
(62)
Facts: Bugay, auditor of the Kapisanan Ng Mga Manggagawa Sa
Manila Railroad Company (Kapisanan) and a payroll clerk of Manila
Railroad.
Bugay was requested by the secretary-treasurer of the company to
deliver certain documents belonging to the union and in compliance
therewith he delivered them without consulting the officers of the union.
Making use of these documents, the management of the company
president of the union a charge for falsification of commercial
document.
Subsequently, charges for disloyalty and conduct unbecoming a union
member were preferred against Bugay, which caused his explusion.
Bugay filed a charge for ULP against the union before the CIR which,
after due hearing, rendered decision holding that Bugays expulsion
was illegal it appearing that the same has not been approved by the
majority of the chapters of the union as required by its constitution and
by-laws. Bugay did not have sufficient opportunity to defend himself.
Kapisanan filed a MTD on the ground that the complaint does not state
facts sufficient to constitute a COA in that neither the decision of the
CIR nor that of the SC contain any statement that the ULP act imputed
to the Kapisanan was true. Further, that majority of the chapters voted
in favor of Bugay's expulsion is not borne by the evidence.
Issue No. 1: WON Bugay expulsion as a union member is proper.
SC: No. Bugays affiliation with the Kapisanan was never
terminated. That being the case, Bugay is entitled to all the rights
and obligations appertaining to every member of the Kapisanan
Union actions, whether favorable or otherwise, must be taken by the
chapters within a period of ten days from the time they receive the
resolution. Even under the assumption that the proceedings against
Bugay were not irregular, the resolution in question never had any valid
effect on his union membership. Considering that he has been unduly
and discriminatorily deprived of such rights and obligations, the Court
finds, and so holds, that the Kapisanan, by their act and conduct, have
engaged in and are engaging in ULP.
It should be observed that the main basis of Bugay's action is his claim
that because of the ULP committed by the officers of defendant union
as found by the CIR and the SC, he has suffered moral damages. It is
true that the decisions both of the CIR and SC do not contain any
statement that the charges preferred by the officers of the union
against him which resulted in his expulsion were "trumped up" or
fabricated, or that said officers acted maliciously or in bad faith, but the
fact remains that the two courts have found that his expulsion was
illegal because of the irregularities committed in his investigation. In
effect, it was found that not only has he not been given an opportunity
to defend himself but his expulsion was not submitted to the different
chapters of the union as required by its constitution and by-laws.
Issue No. 2: WON Bugay was afforded due process.
SC: No. In the investigation held and in the board meeting where the
committees report recommending Bugays expulsion was approved,
Bugay was not present. The committee assigned to summon him failed
to serve notice upon him because he was in Lucena. However,
proceedings still continued anent the absence of Bugay.
ATTY. ALLAN S. MONTAO, petitioner, vs. ATTY. ERNESTO C.
VERCELES, respondent.
FACTS: - Atty. Montao worked as legal assistant of FFW Legal
Center. Subsequently, he joined the union of rank-and-file employees,

15

the FFW Staff Association, and eventually became the employees


union president. He was likewise designated officer-in-charge of FFW
Legal Center.
During the 21st National Convention and Election of National Officers
of FFW, Atty. Montao was nominated for the position of National VicePresident. The Commission on Election (FFW COMELEC) informed
him that he is not qualified for the position as his candidacy violates the
1998 FFW Constitution and By-Laws. Atty. Montao thus filed an
Urgent Motion for Reconsideration praying that his name be included
in the official list of candidates. Election ensued in the National
Convention. Despite the pending motion for reconsideration with the
FFW COMELEC, and strong opposition and protest of respondent Atty.
Ernesto C. Verceles (Atty. Verceles), a delegate to the convention and
president of University of the East Employees' Association (UEEAFFW) which is an affiliate union of FFW, the convention delegates
allowed Atty. Montao's candidacy. He emerged victorious and was
proclaimed as the National Vice-President.
Through a letter to the Chairman of FFW COMELEC, Atty. Verceles
reiterated his protest over Atty. Montao's candidacy which he
manifested during the plenary session before the holding of the
election in the Convention. Atty. Verceles sent a follow-up letter to the
President of FFW requesting for immediate action on his protest.
Atty. Verceles, as President of UEEA-FFW and officer of the Governing
Board of FFW, filed before the BLR a petition for the nullification of the
election of Atty. Montao as FFW National Vice-President. The BLR
rendered a Decision dismissing the petition for lack of merit. While it
upheld its jurisdiction over the intra-union dispute case and affirmed,
as well, Atty. Verceles' legal personality to institute the action as
president of an affiliate union of FFW, the BLR ruled that there were no
grounds to hold Atty. Montao unqualified to run for National VicePresident of FFW. Atty. Verceles filed a Motion for Reconsideration
but it was denied by the BLR. Atty. Verceles thus elevated the matter
to the CA via a petition for certiorari, arguing that the Convention had
no authority under the FFW Constitution and By-Laws to overrule and
set aside the FFW COMELEC's Decision rendered pursuant to the
latter's power to screen candidates. The CA set aside the BLR's
Decision.
Believing that it will be prejudiced by the CA Decision since its legal
existence was put at stake, the FFW Staff Association, through its
president, Danilo A. Laserna, sought intervention. The CA issued a
Resolution denying both Atty. Montao's motion for reconsideration
and FFW Staff Association's motion for intervention/clarification.
Hence, the present petition. -In their brief, petitioner contended the
Court of Appeals seriously erred in upholding the jurisdiction of the
Bureau of Labor Relations over the intra-union dispute as well as the
collateral attack on the legality of FFW as a legitimate labor
organization.

well-settled that FFW, having local unions all over the country, operates
in more than one administrative region. Therefore, this Bureau
maintains original and exclusive jurisdiction over disputes arising from
any violation of or disagreement over any provision of its constitution
and by-laws.
(2) NO, he cannot. To begin with, FFW COMELEC is vested with
authority and power, under the FFW Constitution and By-Laws, to
screen candidates and determine their qualifications and eligibility to
run in the election and to adopt and promulgate rules concerning the
conduct of elections. Under the Rules Implementing the Labor Code,
the Committee shall have the power to prescribe rules on the
qualification and eligibility of candidates and such other rules as may
facilitate the orderly conduct of elections. The Committee is also
regarded as the final arbiter of all election protests. From the
foregoing, FFW COMELEC, undeniably, has sufficient authority to
adopt its own interpretation of the explicit provisions of the federation's
constitution and by-laws and unless it is shown to have committed
grave abuse of discretion, its decision and ruling will not be interfered
with. The FFW Constitution and By-laws are clear that no member of
the Governing Board shall at the same time perform functions of the
rank-and-file staff. The BLR erred in disregarding this clear provision.
The FFW COMELEC's ruling which considered Atty. Montao's
candidacy in violation of the FFW Constitution is therefore correct.
We, thus, concur with the CA that Atty. Montao is not qualified to run
for the position but not for failure to meet the requirement specified
under Section 26 (d) of Article VIII of FFW Constitution and By-Laws.
We note that the CA's declaration of the illegitimate status of FFW Staff
Association is proscribed by law, owing to the preclusion of collateral
attack. We nonetheless resolve to affirm the CAs finding that Atty.
Montao is disqualified to run for the position of National VicePresident in view of the proscription in the FFW Constitution and ByLaws on federation employees from sitting in its Governing Board.
Accordingly, the election of Atty. Montao as FFW Vice-President is
null and void.
MANALAD vs. TRAJANO, 174 S 322
TOPIC: DISQUALIFICATION CANDIDATE
FACTS: The parties are employees of United Dockhandlers, Inc, rival
groups in the Associated Port Checkers and Workers' Union (APCWU)
Petitioner led by Ricardo R. Manalad, with respondent Pablo B. Babula
heading the group of private respondents.
Although qualifications have been earlier questioned, Manalad et al
won the elections for APCWU officers on November 26, 1984. Babula
et al filed petition for review and on July 3, 1985, the court promulgated
a resolution to dismiss petition for lack of merit and have petitioner
Babula et al vacant APCWU offices and turnover management to
Director of the Bureau of Labor Relations, all for immediate execution,
to be followed by a special election to be held on July 20, 1985 (to be
held under the personal supervision of Director Trajano and his staff).
Babula et al were alleged to refuse compliance with the above
resolution as documented in the petition filed by Manalad et al. The
July 20 1985 special election was held having Babula et al as winners
and duly elected officials of APCWU.
Manalad et al filed petition to disqualify Babula et al as winners due to
their non-compliance to the July 3 1985 resolution, but Director Trajano
dismissed their petition and proclaimed Babula et al as the winners of
the July 20, 1985 special elections. Manalad et al, then, filed petition
to SC to reverse resolution of Trajano, have Babula et al disqualified
and annul the July 20 1985 elections/conduct re-elections.
In 1988, when 3-year term for the disputed 1985 election expired, a
new set of officers for ACPWU has been elected despite motion for
RTO. Manalad et al prayed for the annulment of 1988 elections.

ISSUES: (1) Whether or not BLR has jurisdiction over the intra-union
dispute.
(2) Whether or not Atty. Montano can run legally and validly for the
said position.
HELD: (1) YES, it has jurisdiction over such. Section 226 of the Labor
Code clearly provides that the BLR and the Regional Directors of
DOLE have concurrent jurisdiction over inter-union and intra-union
disputes. Such disputes include the conduct or nullification of election
of union and workers' association officers. There is, thus, no doubt as
to the BLR's jurisdiction over the instant dispute involving memberunions of a federation arising from disagreement over the provisions of
the federation's constitution and by-laws. TDCAIS We agree with
BLR's observation that:
Rule XVI lays down the decentralized intra-union dispute settlement
mechanism. Section 1 states that any complaint in this regard 'shall be
filed in the Regional Office where the union is domiciled.' The concept
of domicile in labor relations regulation is equivalent to the place where
the union seeks to operate or has established a geographical
presence for purposes of collective bargaining or for dealing with
employers concerning terms and conditions of employment.
The matter of venue becomes problematic when the intra-union
dispute involves a federation, because the geographical presence of a
federation may encompass more than one administrative region.
Pursuant to its authority under Article 226, this Bureau exercises
original jurisdiction over intra-union disputes involving federations. It is

ISSUE: Whether or not motion for annulment of 1988 elections is moot


and academic
HELD: Yes. The court found the motion for annulment of the 1988
ACPWU elections moot and academic for the ff reasons:
- It is pointless and unrealistic to insist on annulling an election of
officers whose terms had already expired. We must consequently
abide by our consistent ruling that where certain events or
circumstances have taken place during the pendency of the case

15

which would render the case moot and academic, the petition should
be dismissed.
- The court respects the will of the majority of the workers who voted in
the November 28, 1988 elections.
- Contentions of petitioners do not adequately establish the basis for
contempt but respondents have satisfactorily answered the averments
thereon.
- Obtaining the second highest number of votes does not mean that
they will thereby be considered as the elected officers if the true
winners are disqualified.

employees as arbitrary, whimsical, and wanting in legal basis 8 but set


aside the challenged order of July 26, 1986 on the ground that 51 ** of
56 challenged voters were not yet union members at the time of the
election per April 24, 1986 list submitted before the Bureau. 9 The
decision directed among others the proclamation of Lacanilaos group
as the duly elected officers and for ITM-MEA to absorb in the
bargaining unit the challenged voters unless proven to be managerial
employees. 10 Petitioners motion for reconsideration was likewise
denied.
Dissatisfied with the turn of events narrated above petitioners elevated
the case to this Court by way of the instant petition for certiorari under
Rule 65 of the Rules of Court.
Petitioners allege that public respondent director of Labor Relations
committed grave abuse of discretion in ordering the Med-Arbiter to
disregard the 56 segregated votes and proclaim private respondents
as the duly elected officers of ITM-MEA whereas said respondent ruled
that the grounds relied upon by ANGLO for the exclusion of voters are
arbitrary, whimsical and without legal basis.

TANCINCO vs. PURA FERER-CALLEJA, 157 S 2003


FACTS: This special civil action for certiorari seeks to annul the
Resolution and the Decision of the Bureau of Labor Relations setting
aside the order which decreed the inclusion and counting of the 56
segregated votes for the determination of the results of the election of
officers of Imperial Textile Mills Inc. Monthly Employees Association
(ITM-MEA).
Private respondents are the prime organizers of ITM-MEA. While said
respondents were preparing to file a petition for direct certification of
the Union as the sole and exclusive bargaining agent of ITMs
bargaining unit, the unions Vice-President, Carlos Dalmacio was
promoted to the position of Department Head, thereby disqualifying
him for union membership. Said incident, among others led to a strike
spearheaded by Lacanilao group, respondents herein. Another group
however, led by herein petitioners staged a strike inside the company
premises. After four (4) days the strike was settled. An agreement was
entered into by the representatives of the management, Lacanilao
group and the Tancinco group the relevant terms of which are as
follows:
1. That all monthly-paid employees shall be United under one union,
the ITM Monthly Employees Association (ITM-MEA), to be affiliated
with ANGLO;
2. That the management of ITM recognizes ANGLO as the sole and
exclusive bargaining agent of all the monthly-paid employees;
3. That an election of union officers shall be held on 26 May l986, from
8:00 a.m. to 5:00 p.m.;
4. That the last day of filing of candidacy shall be on l9 May l986 at
4:00 p.m.;
5. That a final pre-election conference to finalize the list of qualified
voters shall be held on 19 May 1986, at 5:00 p.m.;
A pre-election conference was held, but the parties failed to agree on
the list of voters. During the May 21, 1986 pre-election conference
attended by MOLE officers, ANGLO through its National Secretary, a
certain Mr. Cornelio A. Sy made a unilateral ruling excluding some 56
employees consisting of the Manila office employees, members of
Iglesia ni Kristo, non-time card employees, drivers of Mrs. Salazar and
the cooperative employees of Mrs. Salazar. Prior to the holding of the
election of union officers petitioners, 2 through a letter addressed to
the Election Supervisor, MOLE San Fernando Pampanga, protested
said ruling but no action was taken. On May 26, 1986, the election of
officers was conducted under the supervision of MOLE wherein the 56
employees in question participated but whose votes were segregated
without being counted. Lacanilaos group won. Lacanilao garnered 119
votes with a margin of three (3) votes over Tancinco prompting
petitioners to make a protest. Thereafter, petitioners filed a formal
protest with the Ministry of Labor Regional Office in San Fernando,
Pampanga 3 claiming that the determination of the qualification of the
56 votes is beyond the competence of ANGLO. Private respondents
maintain the contrary on the premise that definition of unions
membership is solely within their jurisdiction.
On the basis of the position papers submitted by the parties MOLEs
Med Arbiter 4 issued an order dated July 25, 1986 directing the
opening and counting of the segregated votes. 5 From the said order
private respondents appealed to the Bureau of Labor Relations (BLR)
justifying the disenfranchisement of the 56 votes. Private respondents
categorized the challenged voters into four groups namely, the Manila
Employees, that they are personal employees of Mr. Lee; the Iglesia ni
Kristo, that allowing them to vote will be anomalous since it is their
policy not to participate in any form of union activities; the non-time
card employees, that they are managerial employees; and the
employees of the cooperative as non-ITM employees. 6 On December
10, 1986, BLR rendered a decision 7 holding the exclusion of the 56

ISSUE: Whether the order of segregation (exclusion) of 56 votes of the


petitioners union by the respondent is tantamount to grave abuse of
discretion.
HELD: The petition is impressed with merit. The record of the case
shows that public respondent categorically declared as arbitrary,
whimsical and without legal basis the grounds 11 relied upon by
ANGLO in disenfranchising the 56 voters in question. However, despite
said finding public respondent ruled to set aside the Resolution of July
25, 1986 of the Med-Arbiter based on its own findings 12 that 51 of the
56 disenfranchised voters were not yet union members at the time of
the election of union officers on May 26, 1986 on the ground that their
names do not appear in the records of the Union submitted to the
Labor Organization Division of the Bureau of Labor on April 24, 1986.
The finding does not have a leg to stand on. Submission of the
employees names with the BLR as qualified members of the union is
not a condition sine qua non to enable said members to vote in the
election of unions officers. It finds no support in fact and in law. Per
public respondents findings, the April 24, 1986 list consists of 158
union members only 13 wherein 51 of the 56 challenged voters names
do not appear. Adopting however a rough estimate of a total number of
union members who cast their votes of some 333 14 and excluding
therefrom the 56 challenged votes, if the list is to be the basis as to
who the union members are then public respondent should have also
disqualified some 175 of the 333 voters. It is true that under article
242(c) of the Labor Code, as amended, only members of the union can
participate in the election of union officers. The question however of
eligibility to vote may be determined through the use of the applicable
payroll period and employees status during the applicable payroll
period. The payroll of the month next preceding the labor dispute in
case of regular employees 15 and the payroll period at or near the
peak of operations in case of employees in seasonal industries.
n the case before Us, considering that none of the parties insisted on
the use of the payroll period-list as voting list and considering further
that the 51 remaining employees were correctly ruled to be qualified for
membership, their act of joining the election by casting their votes on
May 26, 1986 after the May 10, 1986 agreement is a clear
manifestation of their intention to join the union. They must therefore
be considered ipso facto members thereof Said employees having
exercised their right to unionism by joining ITM-MEA their decision is
paramount. Their names could not have been included in the list of
employee submitted on April 24, 1986 to the Bureau of Labor for the
agreement to join the union was entered into only on May 10, 1986.
Indeed, the election was supervised by the Department of Labor where
said 56 members were allowed to vote. Private respondents never
challenged their right to vote then.
The Solicitor General in his manifestation agreed with petitioners that
public respondent committed a grave abuse of discretion in deciding
the issue on the basis of the records of membership of the union as of
April 24, 1986 when this issue was not put forward in the appeal.
WHEREFORE, premises considered, the petition for certiorari is
GRANTED. The temporary restraining order issued by this Court is
hereby made permanent. The questioned Resolution and the Decision
are hereby set aside for being null and void and the Order of the
Mediator Arbiter is hereby declared immediately executory.

15

Cost against private respondents.

15

Anda mungkin juga menyukai